Mathematics Green Book 45MB

Download as pdf or txt
Download as pdf or txt
You are on page 1of 288

Download more resources like this on ECOLEBOOKS.

COM

For Performance Measurement

ZIMBABWE SCHOOL EXAMINATIONS COUNCIL

GENERAL CERTIFICATE OF EDUCATION


ORDINARY LEVEL

MATHEMATICS
4008

Past Question Papers and Expected Answers

JUNE 1999 - NOVEMBER"2001


Download more resources like this on ECOLEBOOKS.COM
Candidate
CentreNumber Number

Carxndate Name ---:- --,- __

ZIMBABWE SCHOOL EXAMINATIONS COUNCIL


in collaboration with
UNIVERSITY OF CAMBRIDGE LOCAL EXAMINATIONS SYNDICATE
General Certificate of Education Ordinary Level

MATHEMATICS 4008/1 , 4028/1


PAPER 1
Friday 4JUNE 1999 Morning 2 hours 30 minutes

Candidates answer on the question paper.


No additional materials are required.

TIME 2 hours 30 minutes

INSTRUCTIONS TO CANDIDATES
. Write your name, Centre number and candidate number in the spaces at the top of this page.
Answer all questions.
Write your answers in the spaces provided on the question paper.
If working is needed for any question it must be shown in the space below that question.
Omission of essential working will result in loss of marks .
.Mathematical tables, slide rules and calculators may not be brought into the examination roorg.

INFORMATION FOR CANDIDATES


The number of marks is giv~n in brackets [ ] at the end of each question or part question.

FOR ExAMINER'S USE.

This question paper consists of 20 printed pages.


MFK (0729) lPF92115
f) ZSEC 1999 [Turnover
3
Download more resources like this on ECOLEBOOKS.COM
For 2
For
Examiner's
Examiner's
Use
Use

NEITHER MATHEMATICAL TABLES NOR SLIDE RULES NOR CALCULATORS


MAYBE USED IN 'nus PAPER

1 (a) Find the exact value of


(i) 5,4 x 0,06,
(li) 16,5 - 496.

(b) Express 840m as a fraction of 2 .lkm , giving your answer in its simplest form.

Answer (a) (i) [1]

(ii).......................................... [1]

(b)............................................... [1]

2 (a) (i) Express 0,008478 correct to two decimal places.

(ii) Express 0,008478 in standard form.

(b) Write down, correct to the nearest integer, .-v402.

Answer (a) (i) ~ [1]

(ii)................................. [1]

(b) [1]

4008/1/402811 S99
Download more resources like this
3 on ECOLEBOOKS.COM
For
Examiner's
Use
3 Given that 1=- 4,m = - 3 and n =5,jind the value of

(a) Imn,

(J» 31- m,

(c) 2n 2.

[1]
Answer (a) ...•......................· : .

(b) [1]

(c).................................................. [1]

4 Express 2 x 52 + 3as a number in

(a) base five,

(b) base two.

Answer (a) [1]

(b).................................................. [2]

4008/1/4028/1JS99 [Turn over


5
Download more resources like this
5 on ECOLEBOOKS.COM
For
For
Examiner's
Examiner's
Use 7 Use

In _!hediagram, All is the diameter of the circle ABCD. Chords BC and CD are equal and
CAB = 33°. The side CD is produced to E. .

Calculate

(a) BAD,

(b) BCD,

(c) ABE.

~ °
Answer (0) BAD = ;....................... [1]
~ 0
(h)BCD=· [I]
~
(c)ADE=
°
:...................... [I)
........•..

(Turn over

7
Download more resources like this
6 on ECOLEBOOKS.COM
For
Examiner's For
Use Examiner's
8 (a) At thebegining of an examination the clock i6 the examination room was set to read Use

2 p.m. The examination ended at 5 pm.

Find the angle. in degrees, through which

(i) the hour hand turns during the examination,

. (ii) the minute hand turns during the examination.

(b) Calculate the obtuse angle between the hour hand and the minute hand of a clock
at 12.30 p.m.

Answer (a) (i) ,. 0 [11

(ii) 0 [1]

(b) 0 [1]

9 The table shows the results of the games played by the Gweru Giraffes last season and the
points awarded in the League Table.

Won Drawn Lost

Number of games. 20 4 6
r---.
Points awarded
per game x 2 0

(a) Given that they gained a total of 108 points, calculate the value of x.
,
(b) The Chinhoyi Cheetahsplayed the same number of games as the Gweru Giraffes.
They drew 6 games and won twice as many games as they lost. Calculate the number of
points they gained.

Answer (a) ...............•~..~............................... [1]

(b) ...........................•....................... [2]

••• 111J 1«12lIi11SW

8
Download more resources like this on ECOLEBOOKS.COM
For
Examiner's
7
Use

10 The following notice was displayed at an airport:

Temperatures ("C) at Air Zimbabwe International Destinations. Date: 311199.

Destination Minimum Maximum

Frankfurt -11 -3
1

Larnaca 9 20

London -5 0

Rorne : 10 13

(a) At which destination

(i) was the lowest temperature recorded,

(ii) was the greatest range of temperatures recorded?

(b) On 4th January the minimum temperature recorded in Frankfurt was 3.oC lower than
on 3rd January. .

Calculate the minimum temperature in frankfurt on 4th January.

Answer (a)(i) (1]

(ii) (1]

(b) °C (1]

11 All the lengths on a scale drawing are one eighth of their actual lengths.

Calculate

(a) the actual length of a line represented by a line 5.6 em long on the scale drawing.

(b) the area on the scale drawing which represents an actual area of ~% cm,

AIl~\'wer (1I):...................•......................... cin [It

(b) ............•.............................. cm ' (2)

-II1J!iii,.ael"i,'>W ITum oyer

9
Download more resources like this on ECOLEBOOKS.COM
8 For
For Examiner's
Exeminer's
Use
Use U

The circle ACD has centre 0 and radius 13 cm.


The chord ABC is perpendicular to the straight line DOB and has a total length
AC=24cm.

(a) Calculate the length of OB.

(b) Write down, as a fraction, the value of cos ADD.

Answer (a) cm [2]

(b) cos ADD = ~ ~ [1]

,13 Write down a simple geometrical reason why it is not possible to draw

(a) (i) a quadrilateral ABCD with angles 7fr, 107°,500 and 1000,

(6) a triangle LMNwith sides 11,4 em, 5,2 em and 4,7 em.

(b) What are complementary angles?

Answer (a) (i) ...•..•.............................................


~ , ;............. [1]

(ii) ...........•.......................................................................~........................................ [1]

(b) ..- ::..................................... [1]

401J811/402!1111S1JY

10
Download more resources like this on ECOLEBOOKS.COM
For 9 For
Examiner's·
Examiner's
Use
Use
U (8) Write down the order of rotational symmetry of an equilateral triangle.

Answer (a) [1]

(b) the diagram in the answer space, shade the image of ABeD so that the resulting.
0.11
complete shape, has Be as a line of symmetry .

Answer (b)

[1]

(c) On the diagram in the answer space, shade the image which is the reflection of the
shaded shape in the line PQ.

Answer (c)

(I)

ITurn over
11
Download more resources like this on ECOLEBOOKS.COM
For
10 Examiner's
Use

15 Express

(a) 1,75 hectares in square metres,

(b) 2 cubic metres in litres,

(c) 3.85 hours in hours and minutes.

'.

Answer (a) square metres [1]

(b) ~.........•...•.......
litres [1]

(c) ...•..•.............hours minutes [1)

4OOIlI1/402lII11SW

12
Download
.1
For
more resources like this
11 on ECOLEBOOKS.COM For

Examiner's I Examiner's
Use
Use I
I
16 (a) Solve the inequality -7 < 2-3x ~ 5.

(b) Illustrate your answer to part (8) on the number line in the answer space.

Answer (0) [2]

(b) ~~__.____..-_.__........_~__.____..--+ [1]


-3 -2 -1 0 1 2 3 4

17 A man estimates that each side of a square floor has a length of 4 metres, correct to the
nearest metre.

Find the difference between the largest and smallest possible calculated values of the area .
of the floor.

Answer' •..: :........ m 2 . [3].

4IllW 1/4112111 IISW (Tam over


13
For.
Download more resources like this
Examiner's 12 on ECOLEBOOKS.COM For
Exammer's
Use
Use

18 Solve the equations'

(a) 3h + 4 = 13,
5

(b) y(y + 3) -2(y + 3) = O.

Answer (a) h = ~.................................. [2]

(b) y = or [2]

19 Evaluate

(a) 34 -:- 52, giving your answer as an exact decimal,

Answer (a) [2]

(b) _..................................... [2]

4OOIII1I4IJ11!1I1S911

14
Download more resources like this on ECOLEBOOKS.COM
For
., For
Examiner's
Use I"
J3 Examiner's
Use

20 I

Take 1t to be 2.;.
The diagram shows Q pattern used in dressmaking. It consists of six equal circles of radius
3.5 cm inscribed in a rectangle. The sides of the rectangle are tangents to the circles which
touch other circles as shown. r

Calculate

(a)· the area of the rectangle,

(b) the area of the shaded region.

A,,"\er (lI) cm ' [2}

(h) .- em 2 [2J

----- _._. __ .. _ ...-. -_-- ._----_-._ ._---- _." _"-_. -------_. _. __ ._------_. __ ._._ .. _-_._---_._._-_ .... _---

i
I
I
I
I
I
I

4008/114028/1/899
[Turn over
15
Download more resources like this on ECOLEBOOKS.COM
For
Examiner's
15 For
Examiner':
tom tUs«

22
f(x)

f(x) = (lx - 1)(x - 2)(x + 3)

The.diagram shows the graph off(x) = (lx -l)(x - 2)(x + 3).

(a) Find the coordinates of

(i) A,

(ii) B.

(b) The point P(- 2, m) lies on the curve. Calculate the value of ni.

Answer (a) (i) A is ( ) [ II

(ii) B is ( ; ) [I}

(h)m= : [21

4IlORllI402lIIIJS99
[Turn over
17
Download more resources like this on ECOLEBOOKS.COM
For i
Examiner's i 16 For
Examiner's
Use I Use

i 23 A survey is carried out to find the number ~)tminutes each member of a class takes to tiru:sh
I'i a multiple choice test. The dr, 'gram belowj, an incomplete histogram used to illustrate the
results of the survey.
I

I
II
I
I
I
Frequency
density

o 10 20 30 40 50
Time in minutes

Another way to represent the same information is shown below.

Time (I) ,; < t ,._::20 20 <, ( :s; 30 30 < (~40 40 < t ~ 45

Frequency r ]0 16 9

(a) Find the value of x.


,
(b) Complete the given histogram.

(c) Write down the modal time interval.

Ann1'fr (a) x = [1]

(h) (on the diagram) [2]

(c) "........ [1]

---_. __ ..... _---- --'_'_ ._----_._---.- --------------------------

4008/11402811/899

18
Download more resources like this on ECOLEBOOKS.COM
For
Examiner'~
. 17 For
Examine
tJse Use

24 The strength, S, of a beam of metal varies jointly as the square of its depth, D, and the
inverse of its length, L.

(a) Write down a formula for S in terms of D, L and a constant, k.

(b) Given that S = 18 when D = 3 and L :;:5, find the value of k.

(c) Calculate the value of D when S = 20 and L = 6. leaving your answer .in surd form.

Answer (0) S = [2]

(h)k=:; [2]

(c)D= [2]

._ _ --.-.~---.---- _--_. __ ._------

I 4OO8III4028/lJS99

19
[Turn over
Download more resources like this
For
II
on ECOLEBOOKS.COM For·
Examiner's Examiner's
Use Use

25

In,!he diagram, P,i! and R represent points on level ground with PQ = 3Dm, PR = 24 m,
PRQ = 87° and QPR = 4(f. The bearing of Q from Pis 042°.

Use as much of the information given below as is necessary.

(a) Calculate

(i) the bearing of R from Q,

(ii) how far Q is north of P.

(b) R is the base of a vertical mast RT.


The angle of elevation of the top of the mast, T, from P is also 42°.
Calculate the height of the mast RT.

[sin 42° = 0,67; cos 42° = 0,74; tan 42° = 0,90.]

Answer (a) (i) .................................•.......... [2]

(ii) m [2]

(b) m [2]

,"Willi~14U2X1IIS'l'l

20
Download more resources like this on ECOLEBOOKS.COM
For For
Examiner's 19 Examiner's

Use Use

26 in 1m a television set had a marked price of $7370.00.


(a) Mr Mogo paid cash and was allowed 15% discount. Calculate the discount.

(b) Mr Dube bought the television set through a lay-by scheme. In this scheme he paid an
initial deposit of ~ of. the marked price and then 3 equal monthly instalments,
before he collected the television set. Calculate each monthly instalment.

(c) The marked price in 1999was a result of a 10% increase over the marked price of the
previous year. Calculate the marked price of the television set in 1998.

Answer (a) $ [2]

(h) $ .-.. [2]

(c) $ [2]

4008l1/402811/S99 Tfurn over I


21
----------------------------,------------~~----~-----

'---T-iff--
---
Download more resources like this
For
Examiner's .\"
·20 on ECOLEBOOKS.COM For
Examiner's
Use
Use

27 (a) -r··· ····


..·· ,------, : 8j. . .

I· n

l±t±r_=. .~l·.:::L+.::.:::::::. .
1 2 [- 3. 4 10

x y •• ..:.

Table 1

Consider the pattern of the numbers shown in Table 1 above, which is incomplete.

(i) Write down the value of .r.

(ii) Express y in terms of 11.

(b) Table 2 SilOWS th. first Jines of another pattern

Line Terms Sum of terms

1 1 1

2 1,3 4

3 1,3,5 9

4 1,3,5,7 ...............

Tahl~ :2

(i) Complete line 4 in Table 2 above,


(ii) Complete line 7 in Table 2 above.
(iii) Write down the sum of the terms in line 30 of the table.

Answer (a) (i) x = . . .. .. . rIll


(ii) y =............................ 121 '
(b) (i) (in the table) Ii I I
(ii) (in the table) 121 I
(111) r 11
'.- •. _. - ._.•.._. - . - .__ ... _. -_._.- _--." I _-- _... __ -

4OO8/1J40Z&'1IS99

22
Download more resources like this on ECOLEBOOKS.COM
MATHS JUNE. 199£i

4008/1

POSSIBLE ANSWERS

. 324 . 54 2
1. (a) (0 0,324 or -. - (0 11,54 or 11- (b) 5
. 1000 . 100
or equivalent fraction or equivalent fraction

2. (a) (0 0,1 (ii) 8,478x10-3 (b) 20

3. (a) 60 (b) -9 (c) 50

4. (a) 2035 or 203tive (b) 1101012 or 110 101 two

5. (a) (0 8 (ii) 400 (b) Heptagon

6. (a) (i) 1306 or 1.06pm (b) 235

7. (a) 66 (b) 114 (c) 123

8. (a) (0 90· (ii) . 1080 (b) 165

9. (a) 5 (b) 92

10. (a) (i) Frankfurt (ii) Larnaca (b)


11. (a) 44,8 (b) 14

-5
12. (a) 5 (b)
13

13. (a) (0the sum ofthe angles is less than 36oo


(ii) the sum of the two shorter sides is tess than the third side
(b) two angles which add up to 9oo

14. (a) 3 (b)

23
Download more resources like this on ECOLEBOOKS.COM

(c)
'\

I~
~
~ ~ ~
f'

(a) 17500 . (b) 2000 (c) 3hours 51 minutes


15.

-1 ~ x <3 or
16. (a)
-1 sx and x<3 (b)
• 0

-1 0 3

17, 7,11

15 (b) 2or-3
18, (a)

(a) 3,24 (b) 2


:9.
I

(a) 294 (b) 63


20.
2
(a) 40 (b)
21. 3

(a) (i) (-3;0)


C'\
\II, (C,;6j '(b) 2@
22.

(a) 4 (b) rectangle of height1 ,8 and


23.
interval 30<t~ 45
c) 30<t~40

kD
24. a) - (b) 10 (c) 25orf12
L
fjj) . 22,2 (b) 21,6
25. a) (i) 1690 \ ,

(b) 921,25 (c)' 6700


26. a: 1105,5

("\n. 2n-1
27. (a) (i) 19 1,3,5,7,9,11,13=49 (iii) 900
("ii)
(b) (i) 16
EGI c: 4008/1/J99

24
Download more resources like this on ECOLEBOOKS.COM

ZIMBABWE SCHOOL EXAMINATIONS COUNCIL


In collaboration with
UNIVERSITY OF CAMBRIDGE LOCAL EXAMINATIONS SYNDICATE
General Certificate of Education Ordinary Level

MATHEMATICS 4008/2
PAPER 2
Wednesday 9 JUNE 1999 Morning 2 hours30 minutes
Additional materials:
. Answer paper
Geometrical instruments
Graph paper (3 sheets)
Mathematical tables
Plain paper (1 sheet)

TIME 2 hours 30 minutes

INSTRUCTIONS TO CANDIDATES
Write your name, Centre number and candidate number in the spaces provided on the answer paper/
answer booklet.
Answer all questions in Section A and any three questions from.Section B.
Write your answers on the separate answer paper provided.
If you use more than one sheet of paper, fasten the sheets together.
Electronic calculators must not be used.
All working must be clearly shown. It should be done on the same sheet as the rest of the answer.
Omission of essential working will result in loss of marks.
If the degree of accuracy is not specified in the question and if the answer is not exact, the answer should
be given to three significant figures. Answers in degrees should be given to one decimal place.

INFORMATION FOR CANDIDATES


The number of marks is given in brackets [ 1 at the end of each question or part question.
Mathematical tables may be used to evaluate explicit numerical expressions.

This question paper consists of 11 printed pages and 1 blank page.


MFK (0742) QF92116
©ZSEC 1999

25
Download more resources like this on ECOLEBOOKS.COM
2

Section A [64 marks]

Answer an the questions in this section.

1 ()a E va 1ua t e 221 - 53. -;-3'


2
[2]

(b) Tendai is paid $12,40 an hour for a basic week of 40 hours. He is paid $18,60 an hour for
overtime. Calculate his pay for a week in which he works for 48 hours. [3J
,
(c) There are 1500 pupils at Fundo Secondary School. One third of the.pupils are girls.
(i) Find the number of boys.
(ii) ,Given that ~ of the boys are studying Agriculture, express the number of boys studying
Agriculture as a percentage of the total number of pupils at the schook
[4]

(d) Calculate the simple interest earned on $1200 invested at a rate of 15% per annum .for
6 months. [2]

2 (a) (i) Factorise completely ax + x -- 3a - 3.


(ii) Factorise 2x2 - 7x + .3.
(iii) Write down the L.C.M. of

ax + x - 3a - 3 and 2x2 - Tx + 3.
[5]

(b) Given that 2p = ~+ 5~


(i) find the value of p when q = 20,
(ii) express q in terms of p.
[5]

(c) rf . 2
S'ImplV--'---' 3
. x-1 xt-5 [31

26
Download more resources like this on ECOLEBOOKS.COM

ZIMBABWE SCHOOL EXAMINATIONS COUNCIL


In collaboration with
UNIVERSITY OF CAMBRIDGE LOCAL EXAMINATIONS SYNDICATE
General Certificate of Education Ordinary Level

MATHEMATICS 4008/2
PAPER 2
Wednesday 9 JUNE 1999 Morning 2 hours30 minutes
Additional materials:
. Answer paper
Geometrical instruments
Graph paper (3 sheets)
Mathematical tables
Plain paper (1 sheet)

TIME 2 hours 30 minutes

INSTRUCTIONS TO CANDIDATES
Write your name, Centre number and candidate number in the spaces provided on the answer paper/
answer booklet.
Answer all questions in Section A and any three questions from.Section B.
Write your answers on the separate answer paper provided.
If you use more than one sheet of paper, fasten the sheets together.
Electronic calculators must not be used •
.All working must be clearly shown. It should be done on the same sheet as the rest of the answer.
Omission of essential working will result in loss of marks.
If the degree of accuracy is not specified in the question and if the answer is not exact, the answer should
be given to three significant figures. Answers in degrees should be given to one decimal place.

INFORMATION FOR CANDIDATES


The number of marks is given in brackets [ ) at the end of each question or part question.
Mathematicaltables may be used to evaluate explicit numerical expressions.

This question paper consists of 11 printed pages and 1 blank page.


MFK (0742) QF92116
©ZSEC 1999

25
Download more resources like this on ECOLEBOOKS.COM
3

3 Answer this question on a single sheet of graph paper.

The table below shows the nu~ber of pupils from each form who went on a trip to Great Zimbabwe.

Form Number of pupils

1 25
2 16
3 75
4 64
i
(a) Using a 1 em-wide bar to represent each form and a vertical scale of 2 em to represent 10 pupils,
draw a labelled bar chart for the given data. [3]

(b) For these pupils, find


(i) the modal form,
(ii) the mean number per form.
[3]

(c) If two pupils are chosen at random from those who went on the trip, find the probability that
ili~~re~ili~~rm~. m
(d) Given that the total number of pupils at the school was 1080, find, as a single fraction in its
lowest terms, the probability that a pupil chosen at random from the whole school, went on the
trip and was in either Form 2 or Form 4. [2]

Find the value of r for which the matrix ( r 24). , Iar.


4 (a) 1 IS slilgu [2J
,- 4 - I

(b) G·iven hat (31 _ 2)1 (x) (16)7


t y::: '

calculate the values of x and y. [3]

(c) w, X, Yand Z are four points with coordinates (- 2, J), (4,6), (8, I) and (1, - 2) respectively.
----+
(i)
.... _.
Find XY as a column vector.
(ii) Find I~Y I.
(iii) Calculate the gradient of wx.
(iv) Find the equation of the line parallel to WX which passes t~rough Z. .
161
\

27
Download more resources like this on ECOLEBOOKS.COM
4

5 Answer the whole of this question on a sheet of plain paper.


Use ruler and compasses only for all constructions and show clearly all the construction lines and arcs.

(a) (i) Construct a quadrilateral ABeD in whieh AB = 9,0 em, DAB = 90°, Afic = 60°,
AD = 7,5 em and Be = 6,3 em. . [4]
(ii) Measure and write down the size of BCD. [1]

(b) On the same diagram construct


(i) the locus of points that are 5,0 em from B, . [2]
(ii) the locus of points that are equidistant from DA and DC. [2]

(c) Mark and label points X and Y which are 5,0 em from B and equidistant from DA
and DC. [2]

"

4IIIIrVSW

28
Download more resources like this on ECOLEBOOKS.COM
5
6 In a survey,200 people were asked which of the three meats - beef, chicken or pork - they liked. The
results are displayed in the table below

Type of Meat Number of people

Beef 80
Chicken 120
Pork 50
Beef and Chicken 60
Chicken and Pork 46
Pork and Beef 32
All three 28
None of the three . h

Some of this information is shown in the incomplete Venn diagram below.

Beef

Write down the values of


(i) d,
. (ii) e,
(iii) j,
(iv) g,
(v) h;
[5]
(b) How many people liked exactly two types of meat. [1]

(c) Calculate the number of people who did not like any of the meats as a fraction of the total
number surveyed. Give your answer as a fraction in its lowest terms. [2]

4008/.2/S99

29 [Turnover
Download more resources like this on ECOLEBOOKS.COM
6

Section 8 [36 marks]

Answer three questions in this section.

7.

M 7-x
x

x+4 3x+ 1

The diagram shows a rectangle and a right-angled triangle. The lengths of the sides of the rectangle
M are (x +4) em and (7 - x) em. The lengths of .two of the sides of the triangle N are x em and
(3x +1) cm as shown

(a) Write-down an expression. in terms of x, for the area of •


(i) rectangle M
(ii) rectangle N
[2]

(b) Given that the area of rectangle M is twice the area of triangle N, form an equation in x and
shows that it reduces to
2 -
2x -x-14=O. [3]

(c) Solve the equation in (b), giving your answers correct to 3 significant figures. [5]

(d) Write down the dimensions of rectangle-M correct to the nearest millimetre. [2]

30
Download more resources like this
7
on ECOLEBOOKS.COM

8
y~
6
F=
~
~
~-
4
,_
>
C3
,__
... -- A B A~
B

.---l--+
. --+
6

Triangle ABC has vertices A( - 3, 2), B( - 1, 2) and C( - 1, 3).


Triangle A ,B ,C, has vertices A ,(- 2, - 3), B ,(- 2, - 1) and C,(- 3. - 1).

(a) (i) Describe fully the single transformation that maps 6ABC onto LA IB ICI' [2]
(ii) Write Jown the matrix that represents this transformation. [2]
(b) Triangle A BC is mapped onto triangle A 2B2C 2 by a transformation which isrepresented by the
.:..02 - 0)2 .
matrix (
(i) Find the coordinates of A 2' B 2 and C 2' [2J
(ii) Write down the matrix which represents the transformation which maps 6A 28 2C 2 onto
6ABC.- - [IJ
- -(iii) Find the matrix which represents the transformation which maps l\A2B2C2 onto -
6A ,B IC I' (2)

(c) Triangle AJBJCJ has vertices A_1(3,2). B)(5, 2) and C,(8, 3).
Describe fully the single transformation that maps 6ABC onto 6A1B1C1• [3}

(Tum over
4008/2/899

31
Download more resources like this on ECOLEBOOKS.COM
8

9_ (8,
B-

In the diagram, AB is parallel to DE and AE is parallel to PQ.


ACE and BCPD are straight lines and AC = PQ.
(i) .Explain why triangles ABC and EDC are similar.

(ii) Given that AC = iAE_ and that the area of triangle ABC is 2 square units, find the area of
PQEC.
[4]

(_b)

In the diagram, JKLM is a quadrilateral, 1M = 5 m, KM = 8 m and MlK = 50°.


(i) Calculate J KM. [31
(ii) Given also that MKL = 95~.and KL = 11 m, calculate the length of ML. [51

4008/2/S99

32
Download more resources like this
9 on ECOLEBOOKS.COM
10 (a)' Working 8 hours a day, 45 men could do a job in 12 days. How many men, working at the same
rate, working 7~ hours a day, should be employed to do the jobin 9 days? [3]

(b)

15
,, "
,, "
"
"
,, "
,,- - - --." -,"'" .•..
,
,,--- ..•. ",
,
" .•...•.. .•..
r.•..
__ ... ,'J
..•.... "
,,
,
15

Take 11' to be 3,142

A cylindrical hole of radius 3 em was drilled completely through a metal cube of side 15 em in a
direction perpendicular to one of the faces of the cube, as shown above.
(i) Calculate the volume of the metal removed. [2]
(ii) Express the volume of the metal removed as a percentage of the volume of the original
cube. [2]
(iii) Calculate the volume of t~e_remaining metal. [2J
(iv) All the metal removed was melted and made into 45 identical solid spheres. Calculate the
radius of each sphere, giving your answer correct to the nearest tenth o! a millimetre. [3]

[Volume of sphere = 1 'ITr}]

4008/2/S99

33
Download more resources like this
10 on ECOLEBOOKS.COM
/
11 Answer the whole of this question on a single sheet of graph paper

The velocity, v rnls of a ball after time, t seconds ,is given by

v =4 + si-?

The table below gives some corresponding values of v and t.

(s) 0 1 2 3 4 5 6

" (mls) 4 8 10 10 8 4 P

(a) Calculate the value of p. [1J

(b) Taking 2 em to represent 1 second, draw a horizontal axis for 0 E: t E: 6.


Taking 2 em to represent 2 mis, draw a vertical axis for - 4 E: V E: 12.
Draw the graph of v = 4 + 51 - t2 for 0 E: t E: 6. ' (4]
(e): Use the graph to estimate
(i) the time at which v = 0,
(ii) the maximum speed reached by the ball,
(iii) the distance covered by the ball in the first 2 seconds.
[41
(d) (i) By drawing a tangent, find the gradient of the graph when t = 4.
(6) State briefly what this gradient represents.
[3]

400812/599

34
Download more resources like this on ECOLEBOOKS.COM
11

llA_wer the whole of this question on a single sheet of graph paper.

There were x rows of Ordinary seats and y rows of Superior seats for a school play.

(a) Each row of Ordinary seats had 25 seats and each row of Superior seats had 20 seats.
Fire regulations limit the total number of seats to 400.
Write down an inequality which expresses this restriction and show that it reduces to
5x + 4y ~ SO. [2J

(b) There is space in the hall for no more than 18 rows of seats.

The Headmaster insisted that there were at least 5 rows of Ordinary seats.
Write down two further inequalities, other than x;;a: 0 and r > 0, which express these
restrictions. [2J

(c) The point (x, y)represents x rows of Ordinary seats and y rows of Superior seats.
Using a scale of 2 em to represent 2 rows on each axis, draw .r- and y-axes taking values of x and
y from 0 to 20.
.. . ~
Construct, and indicate clearly by shading the unwanted regions, the region in which (x, y) must
~. ~
.
(d) All of the seats were occupied.

Writ~ down the coordinates of the points which represent the largest permitted audience. [2]

(e) . Each Ordinary seat cost 40 cents and each Superior seat cost$l.

Write down, in terms of x and y, an expression for the total income, in dollars. [1]

(f) Find the greatest possible income, given that all the conditions are satisfied. [1]

4008/21899

~5
Download more resources like
12 this on ECOLEBOOKS.COM

BLANK PAGE

4008Il1S99

36 .
Download more resources like this on ECOLEBOOKS.COM

MATHEMATICS JUNE 1999


4008/4028102

POSSIBLE 'ANSWERS

1. (a) 2'!'_ ~ 2
2 5 3

= -25 9
10 M1

, 3
=1,6 or 15 A1

(b) Total pay =12,40 x40 + 18,60 x 8 M2


= $644,80 A1

,(c) 2
(i) No. of boys = "3X1 500 M1
=1 000 A1
1
(ii) , . No. doing agric '_ -x1000
8
1 1000
% of total = - x x 100 M1
8 1500

1
= 8
3 or 8,333% A1

1200x15x6
(d) I = M1
lOOx12

= $90 A1
[11]

37
Download more resources like this on ECOLEBOOKS.COM

2 (a)
.
(I) x(a + 1)-3 (a + 1) or a (x -3) + x -3 M1
'= (x -3) (a + I ) A1

(ii) 2r- 7x + 3 = (2x -I) (x -3) 82

(iii) LCM . = (a + 1)(x -3) (2x -1) 81

(b) (i) 2p=V6x20+5

2P=5 M1
1 5
P == 22"0r 2~5 or 2" A1

(ii) (2p)3 = 6q + 5 M1
6q = 8p3 - 5 M1
Sp3 -5
q= A1
6
2( x + 5) - 3(x-I
(c) Ml
. (x-l)(x+5)

2x+1O-3x+3
= MAl
(x -l)(x + 5)

13-x
= AI[13]
(x-l)(x+5)

38
Download more resources like this on ECOLEBOOKS.COM

39
Download more resources like this on ECOLEBOOKS.COM

39
--- - ----------------------------------------~~------~
Download more resources like this on ECOLEBOOKS.COM

3. Refer to graph
(a) 'Correct rectangular bars 83

(b) (i) Modal tom = Form 3 . - ·81

(ii) Mean = 25+16+75+64


4 M1

= ~45 A1

MI

185
=-orO,I722 Al
1074

. . 16+64
(d) p( F2 orF 4/ trIP) = -1-0-80- Ml

2
=- Al [10]
27

40
Download more resources like this on ECOLEBOOKS.COM

4. (a) 3r-{ -4)x24=O . MI


r=-32 Al

(b) 3x + 2y =16
x -y=7
3x -3y =21 or 2x -2y =14
5y = - 5 or 5x = 30 MI
y=-I Al
x=6 Al

(c) (i) BI

2
(ii) IXYI= J42 +(-5) Ml
= 6,403 Al

6-3 .1
(iii) grad of WX == - BI
4-(-2) 2

(iv) Equationof line is

y--2 = ..!.(x -1) MI


2
2y=x-5 AI[ll]

41
Download more resources like this on ECOLEBOOKS.COM

B
A

\
\ \
\

42
Download more resources like this on ECOLEBOOKS.COM
5. (a) (i) Refer to diagram
'BAD =900±20 with correct construction arcs
seen C1
"

(,:, ~'t''''.:
,\'l..t·
ABC = 60 ±20 with
o
correct construction arcs
seen. C1

Quadrilateral ABCD with AB =


9±O,2 cm,
AD= 7,5±O,2 cm,BC = 6,3 ±O,2 em,
BAD = 90o±20 and ABC = 60o±20 Q2

(ii) BCD = 139°±2° B1

(b) (i) Complete circle centre B· M1


Correct circle centre B radius 5±O,2cm A1

(ii) Ruled bisector of AC at least 3cm long B1


Correct construction arcs B1

(c) Points X and Y labelled at intersection of


circle and bisector B1 B1
[11]

6. (a) (i) , d = 32 B1
(ii) e = 18 81
(iii)
(iv)
f == 16
0
81
B1
g
(v) h = 60 81

(b) 4+32+18=54 B1

60
(c) MI
200

3
AI[8]
10

7 (a) (i) Area of M = (x + 4) (7 -x) 81

.1
(ii) Area of N ~ 2 x(3x + 1) BI

(b) (x +4)(7 - x) = 2(tx)(3x + 1) MI


7x-x2 -4x+28=3x2 +x
3x2+xr+x+4x-7x-28=0 MAl
4x2 - 2x - 28 = 0
2X2 -x-14 =0 Al
)

43
Download more resources like this on ECOLEBOOKS.COM
Alternatively by completing the square

(x-fi 81
.JW or2,66 81

x=t±2,66 81

-b±.Jb2 -4ac
(c) x=
2a
I±~(-I)'l -4(2)(-14)
2x2
1±.JiT3
= BIBI
4
BI
x =2,91or -2,41 BIBI
=2,91or-2,41 8181

(d) Length = x+4= 6,9 81


Width = 7-x=4,1 BI
[12]

8. (a) (i) Rotation M1


90° anticlockwise about the origin or
2700 clockwise about the origin A1

(ii) (ac dbX-3-1-1


2 2 3
)=(-2-2-3
-3-1 -I
)

44
Download more resources like this on ECOLEBOOKS.COM

-3a + 2b =-2

- a + 2b = -2 :. a = 0 and b =-1

-3c +2d =-3

-
- c + 2d = -I. c = I and d=O

1
.. . Matrix is (~ -0 ) 82

(b) (i) (-2o 0X-3-I-1


2 2 2 3
) M1

Al

(ii) Inverse of - (- 2 0) 1(- 2 0) =-


. 0-2 4 - 0-2

or- -1(1 0) 81
2 °1
(iiO a bX6 2 2) - (- 2- 2- 3 )
-( c d -4-4-6 - - -3-1 -1
M1

6a - 4b =-2
2a -4b =-2 :. a =
6c - 4d = -3 _
° and b = Y2
2c -4d= -1 :. C = -t and d=O

'
:. matrix = (0
_.1
2 -
tJ
0
Al

(c) Shear M1
x -axis invariation A1
shear factor 3 A1

45
Download more resources like this on ECOLEBOOKS.COM

(ii) 424,2 xlOO MI


15x15x15
= 12,6% Al

. .
(iii) Volume of metal remaining =15x15x15 -424,2 M1

=2950,8 cm3 A1

(iv) Volume of one sphere =ltx3x3x15 M1


45

4 3" 1tx3x3xI5
-1tr = ----'--"-- Ml
3 45

r = l,31cm Al

47
Download more resources like this on ECOLEBOOKS.COM

48
Download more resources like this on ECOLEBOOKS.COM

11 Refer to graph

Car p =-2 81

(b) The 6 given points together with (6-2) correctly'


Plotted. ' P3

Correct smooth curve, not grossly thick passing


through the 7 correct points : C1
(e) (i) v = 0 when t =5',7 ±D, 1 secOnds 81
(ii) Maximum speed = 10,3±D,2 mls" 81

(iii) Distance covered in first 2 seconds is


15,3±o,2 m 82

(d). (i) Correct tangent drawn at x =4 , MI


gradient = -3±O,2 A1
(ii) retardation/deceleration/decreasing
speed. 81

[12]

49
Download more resources like this on ECOLEBOOKS.COM

20

18

16
II.,~

-+-

14

--t-
12 -;_Lf-----"--- - ----1--
~ t-- +: _t--- -:_ 4', -lJ
- t---
I I _._-'
f-~
---- _.
10 +-.-~t-+ ;
:J.-f-1 -,+
, , .

··i ,l;
L()
II
6 ~

, I

,t

o 2-41-;+: 4
,t'-+,

50
Download more resources like this on ECOLEBOOKS.COM
. 12. Refer to.graph

(a) 25 x + 20 y ~ 400 Ml

Correct reduction to 5x + 4y ~ 80 A1

(b) x + y ~ 18 BI
x~5 Bl

(c) Line 5x +4y =80 correctly drawn L1


Line x + y =18 correctly drawn L1

Line x = 5 co~ctly drawn L1

Correct region bounded by solid lines R1

(d) (8;10) or (12;5) 82

40
.(e) -x25x+ 1 x20y
100
=10x+20y BI

(f) Greatest income = 8 x10+10 x 20


=$280 81
[12]

~0281299

51
Download more resources like this on ECOLEBOOKS.COM

52
Download more resources like this on ECOLEBOOKS.COM
Candidate
Centre Number Number

Candidate Name _-----'- _

ZIMBABWE SCHOOL EXAMINATIONS COUNCIL

General"Certificate of Education Ordinary Level

: c' .<4\.~.
MATHEMATICS 4008/1 , 4028/1
, PAPER 1 :",: -!if:""-,;

Wednesday 10 NOVEMBER 1999 2 hours 30 minutes

TIME 2 hours 30 minutes

INSTRUCTIONS TO CANDIDATES

Write your name, Centre number and candidate number in the spaces at the top of this page.
Answer all questions.
Write your answers in the spaces orovided on the question paper.
If working is needed for any question it must be shown in the space below that question.
Omission of essential working will result in loss of marks.
Mathematical tables, slide rules and electronic calculators may not be brought into the
examination room .

. INFORMATION FOR CANDIDATES

The number of marks is given in brackets [ J at the end of each question or part question.

FOR EXAMINER'S USE

This question paper consists of 1C) printed pages and 1 blank page.
MFK (0653) QK93954
.:!;) ZSEC 1999 [Turn over

53
I~------_~ ------~~ ------~----------
Download more resources like this on ECOLEBOOKS.COM
For For
Examiner's
Use
2 Examiner's
Use

NEITHER MATHEMATICAL TABLES NOR SLIDE RULES NOR CALCULATORS


MAY BE USED IN THIS PAPER.

1 Express O,O'Z2

(a) as a fraction in its lowest terms,

(b) as a percentage,

(c) in standard form.

Answer (a) r: [1]


.r: ••••.••••••••••••.•••••••.••.•.•.

(b) [1]

(c) [1J

2 Find the value of

(a) 5,08 + 0,946,

(b) v'0,0081,

(c) 5,6 -7,5.

Answer (a) "......... fl]

(b) [11

(c) , [1]
"-.

54
Download more resources like this on ECOLEBOOKS.COM
For
For
Examuu ..-rs
3 Examiner's
U"f Use
3 (a) Simplify 2m(3m +- n) - Srn 2.

(b) Given that f(x) = ~- 5x,find


(i) f( 4),

(ii) f(7a).

lswer (a) [1]

(b)(i) [1]

(ii) [1]

4 p Q

A B C

In the diagram, the lines AB(' and PQ R are parallel. The triangle AQB is equilateral.
Given that QRB = 37°, calculate .

(a) RBC,

(bJ AQR,
(e) QBR.
Answer (a) RBC = ,................... [1]

(b)AQR= [I]

(c)QBR= _ _.. [1J


--------_._------
[Turn over
55
Download
j-,.,
more resources like this on ECOLEBOOKS.COM For
:'.ttlf!linrr's
4 Ez4irIine,'s
15;; Use

5 ABeD is a rhombus whose diagonals meet at O.

State

(a) the number of lines of symmetry of the rhombus,

(b) the order of rotational symmetry of the rhombus,

(c) the size of ADB.

Answer (a) [1)

(b) [1]

(c) [1]

6 A solid rectangular block measuring 6 m x 5 mx 2 m is made up of metal whose density is


7850kglm3• .

(a) Find the IID:tSS of the block in tonnes.

(b) Convert 7850 kg/m 3 to g/cm 3.

Answer (a) tonnes [2]

(b) ; g/cm ' [1]

4OO8I1IW9<I

56
Download more resources like this on ECOLEBOOKS.COM
For 5
. Ezam/Mr',
, Use

7 The cost of 5 rulers is $11,25. The cost of 3 rulers and a pen is $9,35.
Calculate the cost of

(8) a ruler,

(b) a pen.

Answer (a) $ [1]

(h)$ (21
-----------.---------.---------~--------
8 The minimum temperatures recorded in degrees Celsius on six consecutive days were as
follows:

6, 3, -2, 6, -1, 0,
(8) Write down
(i) the lowest temperature recorded, '

(ii) the modal temperature.

lb) Calculate the median tempc~aturc.

/ll/.\'II'~'" (a)(i) , ' ,..' "oC III

(IiI ' °C III

(b) ;....... .. °C II
j
..
, ._'----_ __ __ ._-_.,-_
.•. . _--,-,-
•.. ---
.
,.•.••
rn u,,'r ' I
57
Download more resources like this on ECOLEBOOKS.COM
Fo 6
EXfllllinr"~
Use
9 Solve the simultaneous equations

3x + 2y = -14.
3x-5y ~56.

e:
Answer x = .

y = [3]

10 The exchange rate On a certain day was 3,8 dollars for 1 rand.

Calculate the equivalent of

(8) 150 rands in dbllars,

(IJ) 304 dollars in rands.

Answer (a) $ ..................•................it]

(b) rands [2]


------.---.
",.IKII1WW

58
Download more resources like this on ECOLEBOOKS.COM
7

U Solve the equation x2 - 5x + 1 = 0, giving your answers in surd form.

Answc..r x = ............•.............
cr ~......... [3]

12 In the diagram, Q is due north ol_R. •... f' North


PQR is an isosceles triangle with PQ = 1;/\ :lOdQPR = 36°.

(a) Calculate, giving each answer in three-figure notation,


(i) the bearing of Q from P.
(ii) the bearing of P from R.

(b) Calculate reflex QPR.

Allswer (a)( i) [I J

(.ii) (t1

(b) reflex QPR r II


--_._._ ...

1'1,••oJ uH'r

59
Download more resources like this on ECOLEBOOKS.COM
For 8 For
Examiner's Examiner's
Use Use
13

In the diagram, ABeD is a cyclic quadrilateral.


DC produced meets AB"produced at E..•...
Given that BC = BE, ABC = 840 and DAC = 1<)0, calculate

(8) BCE,

(b) ADC,

(c) CAB.

Answer (a) BCE = :.......................... [1]

(b) ADC = (1J

(c) CAB = [1]

14 (8) (i) Solve the inequality 3x - 5> 21.


(n) Write down the smallest integer value of x for which 3x - 5 > 21.

(b) Illustrate the inequality -4 < x ~ 2 on the number line shown in the answer space
~~ .

Answer (a)(i) (1)

(ii) [1J

40U1!I1IW'1Y

60
Download more resources like this on ECOLEBOOKS.COM
For .
Eumbttr', 9
Use
15 The s~e on a map is such that 6 em on the map represents 1,5 kID on the ground. Calculate

(8) the length" in kilometres, of a road which measures 42 em onth~ map,

(b) the area on the map, in square centimetres, that represents alake of area 8 km2•

Answer (a) : 4- •••••••••••••••••••••• km [1]

(b) ................................•...........cm2 [2]

16 (8) Evaluate 2345 + 1425, giving your answer in base 5.

(b) Subtract 28 minutes 27 seconds from 58 minutes 4 seconds, giving the answer in
minutes and seconds.

(c) Convert 2 days, 6 hours and 27 minutes to minutes.

I'
i
i
I

Answer (a) , :, [IJ

(0) min ..: s JI)

(c) min [1]

4(~IHII1W'!'l [Turn over

61
Download more resources like this
10 on ECOLEBOOKS.COM
For For
Examiner's Examiner's
Use Use

17 Find the value of

(a) 72+ 4 0,

(b) ~6x27.

Answer (a) ........•.......................................... [1]

(b) [2]

4008/11W99

62
Download
For
more resources like this on ECOLEBOOKS.COM
Examiners 11 For
Examiners
Use Use
18

S (12,10)

8 P
(4,8) .

4
.-'-

-~ H··t
o 10 12 14 16 X

,
The diagram above shows points P( 4,8). Q(9, 3) and S( 12. 10).
~
(a) Write down PQ in column vector form.

(b) Mark and label or the diagram

(i) the point R such that PQRS is a parallelogram.


(ii) the point T. the image of S. under a clockwise rotation of <JO°about P.

,Inn..,',. (a) I'Q ~ (


) "'
(h)(i) on diagram III

(ii.) 011 diagram


~ .
(II
-----_. __ -_._-_.-._- __ __
. ._---_. __ _ ------
--.- .. .. ---- -- .. -.-_ - --.-.-

[Turn over

63
Download more resources like this
For
Exominer's
12 on ECOLEBOOKS.COM
Ust:
19 '(a) Measured correct to the nearest centimetre, the sides of a kite are 15 em and 12 cm.
Find the smallest possible perimeter of the kite.

.
(b) " E snmate, . ifi cant figure, t.heva Iue 0f 00627'
correct to one sign 94,6
,

Answer (a) : ;.. em (2)

(b)................................................... [2J

20 There are 500 pupils at a school.


Given that one in every four pupils rides to school, calculate

(8) the number of pupils who ride to school,

(b) the probability that


(i) a pupil chosen at.random does not ride to school.
(ii) two pupils chosen at random ride to school.

Answer (a) [1]

(b)(i) [1]

(ii) [2]

4OIIII11W99

64
Download more resources like this on ECOLEBOOKS.COM
For
For
Ezominer's
13 EmmiMr's
Un u"
21 . Two of the angles of a pentagon are 3xo and llxo respectively. Their sum is 168°.

(8) Calculate the sizes of the two angles.

(b) Given that the remaining three angles are in the ratio of 3: 4: 5, calculate the size of
the largest of these angles. .

/\ll.\'Jl'l'r (a) ...........•......................................: (1)

................................................... [1]

(h) [2J

2 .:'
22 (a) Given that - -.-- -- exprcssv in terms of I' and q.
x -'3" ,/

5111 2111 +3
(b) Express as a single f, action in its simplest form.

,.\11.\11'('" (iI).\ ; . . 121

(h t : ·..·..1 2t

j'J'urnnver

·65
Download more resources like this on ECOLEBOOKS.COM
14

23· A

In the diagram DOC = 90", ADB = BDC, AD = 15 em, DB = 8 em and DC = 10 em.


(8) Calculate BC.

(b) Find cos BDC~

(c) Calculate AB2.

-
Answer (a) Be= em [1]

(b)cosBDC= : [1]

(c) AB2 = cm2 [2J

-v,

66
Download more resources like this on ECOLEBOOKS.COM For
For 15 EutniMr'$
FJlUltiltr,'s Uu
Uu

24 (a) H the matrix 30 0 ) represents an enlargement, with the origin as centre,


( 15
0 x
4
find the value of x.

(b) The matrix (i~ ~) is singular.

Calculate the two possible values of y.

Answer (a) x = [2]

(I» Y = or [3]

4008111W99 [Turn over

67
Download more resources like16 this on ECOLEBOOKS.COM
For For
Examinfr's ~"s
U<e • Use
2S A bus left Bulawayo for Kwekwe with 60 passengers. It passes through Gweru (its only
stop) where it picked up 27 passengers and dropped off n passengers.

(a) Write down, in terms of Il, the number of passengers in the bus as it left Gweru.

(b) Given that it arrived in Kwekwe with 2n passengers, form an equation in n and use it to
find the value of n.

(e) Did the bus gain or lose passengers in Gweru? Explain your answer.

Answer (a) r IJ

(b) Equation " [lJ

n= [JI

(c) [11

Explanation: : .

I
1
I

III I
-_._ .._------'---;,.--.-------- 1
'1
4008/11W99

68
Download
tor more resources like this
17
on ECOLEBOOKS.COM For
"':.ltlmiflf'·.\" Exumintr.\·
Ii,.. u.'<!
26 A cyclis: starts a30 krnjourney at09 OO.She maintains an average speed of20 km/hfor the
first th rcc-q uartcrs of an hour and then rests. Subsequently she continues her journey at an
average speed of 25 krn/h. arriving at her destination at 11 00.

(a) Calculate the distance covered in the first three-quarters of an hour.

(b) Calculate. in hours. the time taken to cover the last part of the journey.

(c) Calculate. in minutes. the duration of her rest.

'(d) The graph shown in the answer space is an incomplete distance-time graph for the
cyclist. Complete the graph.

I
I

I
!
.411sl\'cr (0) km [1]
.1
i

I (h) h [1]

I
I
(c) min [1J
I
!
II . (d) on graph (2]
t

Distance
(km)

. '
I-Hf-H-hI''++++H.· ,. -+-+ ; t .
H·ti: ITt· + +++ ...+ ,...H+++t+
H-#++t++-i-+++++
t- ..

i ~ -+
'

+ t t· .
1B~t1t~ I I
-r-t

0900 1000 1100 1200


I
Time (II)

[Turn over

69
Download
1-"", more resources like this
18
on ECOLEBOOKS.COM For
Fxumincr» f:.tomill(f 's
1/ ••
·,· Usr
,
27 'It is given that log 1050 = 1,6lJXlJ7 correct to five decimal places.

(3) Write down log 1050 correct to 4 decimal places.

(b) Evaluate. giving each answer correct to 4 decimal places.


(i) log 1115.:
(ii) log 1t125.

(c) Evaluate. giving the answer correct to 3 decimal places, log H~'

Answer (a) :............................... [1]

(b)(i) ~...................................... [1]

(ii) [1]

[2]
(c) ..~................................................

70

I
Download more resources like this on ECOLEBOOKS.COM
For 19- For'
Examiners
Examiners
Use Use

28 B

/
/
/
/
/
/
/
/
/
/
/
/
/
/-
/
/
/

. /
/
/

A s c

In this question take IT tobe 3,14.

The diagram shows the right angled triangle ABC


The sector AQRSis drawn inside it such that BRC is a tangent to the sector at R
Given that AB =15cm, AC = 20 em and BC =25 em, calculate

(a) the area of the triangle ABC,

(b) the radius, AR,of the sector AQRS,

(c) the area of the shaded region.

Answer (a) cm [1]

(b) AR= em [2]

(c) em [3]

4oosilW99

71
Download more resources like this on ECOLEBOOKS.COM
20

BLANK PAGE

72
Download more resources like this on ECOLEBOOKS.COM
MATHS NOVEMBER 1999

4008/1

POSSffiLE ANSWERS

2 1
1. (a) - 9 (b) 7,2% or 7-% or 7-% (c) 7,2 X 10-2
125 10· 5

, 26 9 9
2. (a) 6,026 or 6- (b) 0,09 or - (c) -1 9 or -1 -
1000 10 , 10

6~
. 500

(b)(i) -17 (ii) 3-35a .

4. (a) 37° (b) 120° (c) 83°

5. (a) 2 ortwo (b) 2 or two

85 17
6. (a) 471 (b) 7.85 or 7- or 7-
100 20

7. (a) 2,25 (b) 2,60

. 1. 3
8. (a) (i) -2 (ii) 6 (b). I-or 1,5 or-
2 ' 2

9. (a) x = 2 and y = -10


10. (a) 570 (b) 80

11. 5 + 51 .or 5- 51
2/ 2

12. (a) (i) 072° '(b) 288° (c) 324°

13. (a) 42° (b) 96° (c) 23°

,)
14: (a)(i) x > 8 2
3,
(ii) 9 (b) 0 e·
I .
-4 b 2
1
15. (a) 10- or 10,5 (b) 128
2

16. (a)431s0r431s (b) 29 min37s (c) 3267

73
Download more resources like this on ECOLEBOOKS.COM
17. (a) 50 (b) 12

18. <a) ( _ ~) (b) R(17:5) (c) T (6;0)

19. (a) 52 (b) 2000

20. (a) 125 . '(b)(i) !or 0,75 (ii) 31


11 499

21. (a) 36° ;132° (b) 155°

22. (a) 2q ~ 15p (b) m - 6


8

23. (a) 6' (b) 1: or 0,8 (c) 97


5

24. (a) 8 (b) 6 or-6

25. (a) 87- n (b) 87-n = 2n. (c) It lost.


n=29 Picked up 27 and
dropped off 29.

3 (c) 39
26. (a) 15· .(b)"5 orO,6

" (d)

30 -; -; ,

15·····
..·.·.·········
..·....····\
,

,
1100
..
0900 0945 1024

27. (a) 1,6990 (b) (i) 0,6990 (ii) 1,3 979 (c) 0,301

28. (a) 150 (b) 12 (c) 36,96

A:4008-I-n99cs

74
Download more resources like this on ECOLEBOOKS.COM

ZIMBABWE SCHOOL EXAMINATIONS COUNCil


General Certificate of Education Ordinary level

MATHEMATICS 4008/2,
PAPER 2
Thursday 11 NOVEMBER 1999 2 hours 30 minutes "",. "'§

Additional materials;
Answer paper
Geometrical instruments
Graph papel't2 sheets)
Mathemattell tat>1~ 4'" ,""

"'" <~~'~~!Jlf~~etrf'!<, "'",~",


,£:';_~¥:;""~
~:< ..",,:,:o> 'iJ.r:>{f/"" ,~"."" ;2

TIME ' 2 hours 30 minutes

INSTRUCTIONS TO CANDIDATES

Write your name, Centre number and canoidate number in U18 spaces provided on the answer paper/
answer booklet.
Answer all questions in Section A and any three questions trorn Section B.
Write your answers and working on the separate answer paper provided.
If you use more than one sheet of paper. fasten the sheets together.
Electronic calculators must not be used.
All working must be clearly shown. It should be done on the same sheet as the rest of the answer.
Omission of essential working wi!! result ir; loss ot marks '
If the degree of accuracy is not specified In the question and if ft,£; answer is not exact. the answer should .
be given to three significant figures. Answers in degrees sh~!1I!0be qiven to one decimal place.

INFORMATION FOR CANDIDATES

The number of marksis given in brackets [] at the end of each question or part question.
,Mathematical tables may be used to evaluate t?Xi)ic:l numerical expressions

[Turn over

75
Download more resources like this on ECOLEBOOKS.COM
2
(

Section A [64 marks]

Answer all the questions in this section.

"1 3 1
1 (8) Evaluate 5 3 x 4: - 2" ~ [2]

(b) Given that p = 3w2 - xy, find the value of p when w = - 2, x = 6 and y = - 3. [2]

(e) During a sale the price of a radio was reduced by 28% to $486.
Calculate the original price of the radio. [3]

(d) When $4400amounts to $5786in 18 months; find the rate of simple interest per annum. [4]

2 <8, Factorise completely


• (0 3a - ab - 2b + 6, [2]
(ii) 3x2 - 147. [2]

y 1 5
(b) (i) Show that the equation -- - - = -- . reduces to
y-l 3 y+l
2 .
Y - 6y + 8 ={). [4]

(ii)' Solve the equation y 2 - 6v + 8 = O. [2]

4IlO8I2IW9\I.
76
-~~-~--------------~,-,-----
Download more resources like this on ECOLEBOOKS.COM
3

(a) In the diagram, TB is a diameter of the circle. The chords TA, TC and TD are drawn so
that ATB = BTC = CTD;: 25°,
(i) State the reason why AB = BC = CD. [1]
(ti) Calculate
.-
(a) ABT, [1]
""
(b) TeD, [2]
(c) ADT.
.- [2]

(b) In a semicircle of radius 17 em two chords are drawn parallelto the diameter. Given that the
chords are of lengths ~ em and 16 em, calculate the perpendicular distance between them. [3]

4 (a) It is given that OP = (!), PQ = (~2)and oQ = (~~).


(I) Find the value of a and the value of b. [3J
(ii) Calculate lOP--- I. [1J

(b) _Answer the whole of this question ona sheet of plain paper.

Use ruler and compasses only. All construction lines and arcs must be clearly shown.
(i) Construct, on a single diagram,
(a) a triangle XYZ in which XY = 9,5 em, YXZ = 45° and XZ = 6,3 em, [3J
(b) the locus of points equidistant from X and Y, [11
.
(c) the locus of points equidistant from Yand Z, [1]
(d) the circle passing through X, Yann. [2]
(ii) Measure and write down the radius of the circle in (bJ(i)(d). [1]

[Turnover

77
Download more resources like this on ECOLEBOOKS.COM
4

[2]

(b)

In }he diagram, ABeD is a trapezium in which, DC = 6,4 em, BD = 9 em, AB = 2DC and
BDC=34°.

Find
(i) ABD, [1]
(ii) the area of triangle ABD. [3}

(c) The table below shows part of Mr Banda's telephone bill for January.

January monthly rental $ .........................


Cost of metered calls from 010561 to 012777 $ 886,40
(................ units at ................ cents per unit.)

Sub total $ 910,40


Sales tax at 17~% $ .........................

Total amount due $ .........................

Calculate
(i) the January monthly rental, [1J
(ii) the number of units used, [1]
(iii) the charge per unit in cents, [1J
(iv) the sales tax, [1]
(v) the total amount due. (1]

4008/2JW99

78
Download more resources like this
5
on ECOLEBOOKS.COM

6 (a) An agriculture class picked tomatoes.


x pupils picked more than 49 tomatoes,
. 3x pupils picked fewer than 51 tomatoes and
5 pupils picked exactly 50 tomatoes.

It is given that

~ = the set of pupils in the agriculture class,

A = the set of pupils who picked more than 49 tomatoes,

B = the set of pupils who picked fewer than 51 tomatoes.


(i) Illustrate this information on a clearly labelled Venn diagram. [3]
(ii) Given that the total number of pupils who picked tomatoes was 47. find x. [2]

(b) There are 30 red balls and x yellow balls in a bag. A ball is drawn at random from the bag.
(i) Write down, in terms of x, an expression for the probability that the ball drawn is
yellow. [1]

(ii) . Given that this probability is ]_ . find x. [2]


13

(c) The sets P and Q are such that n(P) = 0, lit (_)) =- l) and P r-. Q * 0. Find
(i) the smallest possible value of n(P u Q), [1]

(il) the largest possible value of II(P u Q). [21

4008l2W99 [Turn over

79
Download more resources like this on ECOLEBOOKS.COM
6

Section B [36 marks]

Answer three questions in this section.

Each question in this section carries 12 marks.

7 (a) y

In the diagram, 0 is the origin. PQ is a straight line with a gradient of -3.and Q is the
point (4, 0).

(i) Find the coordinates of P. . [1J


'(ii) PQ is now reflected in the y-axis to the line PR (not shown), R being the. point where the
line meets the x-axis.
Find the equation l,f PRo [3]

(b) 3
( 01 - ·1). IS t hematri
e matnx representmg. a trans formati
ormation V an d ( _ 21 ~) is the matrix representing a

transformation W.
(i) Calculate the coordinates of the image of (3, -2) under the transformation VW. [3]
(ii) Find the coordinates of a point whose image under the transformation W is (1, 2j. [3]
(iii) Describe completely the transformation V. [2]
---------------~----- - ---------~----------
.

80
Download more resources like this on ECOLEBOOKS.COM
7

8 Answer the whole of this question on a sheet of graph paper.

Using a scale of 2 em to represent 2 units on both axes; draw the x and j-axes for the ranges
o :e;; x :e;; 14 and - 2 :e;; y :e;; 16. .

(a) Show the region in which all the inequalities given below are satisfied, by shading the
UNWANTED regions.
(i) y ~ 2,
(ii) y:e;; 2x,
(iii) 2y -x ~ - 2,
(iv) 5y + 8x :e;; 80.
[8] ,

(b) Using the unshaded region in (a) calculate /

(i) the maximum value of the expression 5x - 3y, [2]


(ii) the minimum value of the expression 5x - 3y. [2]

9 , Three points L, M and N are on horizontal ground with M due east of L.


The bearing of N from L is 048° and from M is 325 0

LN= 11 km and MN= 9km .

. (a) Calculate
(i) LNM, [2]
(ii) LM. [4}
I
(b) At midday Peter set out to walk from L towards N.at an average speed of 5 kmlh.
Calculate the time at which he arrived at N. . {3]

(e) At N he changed the direction of his walk to due east until he reached a point D which is due
north of M..

Calculate DM. (3)

81 .
Download more resources like this on ECOLEBOOKS.COM

(iii) .Area of /I gram =APST + t""p J?QRT

=APST +ll.VPQ-ll.VRT

M1

=36+64-4 M1

=96crnZ Ai
[12]

99
Download more resources like this on ECOLEBOOKS.COM

100
Download more resources like this on ECOLEBOOKS.COM

(a) Correct corresponding densities


3; 2; 5,5; 4; '1,5; 1 83 .

Rectangles with centres and heights as in


Table (see graph) H3

(b) Table, with correct values.

Mark I x-45
35 ~10
45 o o·
55 10 400
70 25 750 83
Total = 387,5 81

Mean = 45 + 387,5 M1
200

=46.9 A1

[12]

12 (a) (i) x aL2


Z

x
=ky HI
- 2
Z

(ii) k =-
y
15
=
)

,
=5 81

101
Download more resources like this on ECOLEBOOKS.COM

(iii) Z fkY
,= v-;
= /S-x-2-9-4
·V- 30
MI
=·±7 Al

b) (0 0=25 81

b = 16 81

c =41 81

d=15 82

(ii) t = {n - 1)2 + ,jl M2


=2n2 - 2n + 1 A1
[12}
AM
~NIII

102
Download more resources like this on ECOLEBOOKS.COM
"j;altdk1ate,
• 't;~~}~~;).,:.,

.Candidate~amef'" .,. ".' " A~,.:,.':.:,'-1'


zIMBABM _, ,
SCHOOL EXAMINATIONS
'GeneraI'Certifieate-:of Ed_Cation Ordiaary Level ..
., .. ~,
couNcIL
N •. _ •_ - , _.

MATHEMATICS
PAPER 1

Friday 2 JUNE 2000 Morning 2 hours :30 :iD.inu~


," .- .rf'.'

Candidates answer-on the ques~ pItper.


No additiontltrnfiteritds~. mtuited ..

TIME 2 hours 30 minutes

INSTRUCTIONS TO CANDIDATES

Write your name, Centre number and candidate numberin the spaces at the top of this page.
Answer all questions.
Write your. answers in the spaces provided on the question paper.
If working is needed for any question it must be ShOW1) in the space below that question.
Omission of essential working will result in loss of marks.
Mathematical tables, slide rules and calculators may not be brought into the
'examination room.

IN:FORl\fA TION FOR CANDJI)\ TES

The number of marks is given in br'tck'.'t~ : Iat the end of each question or part question

FOR EXAM1NER'S USE

This question paper consists of 23 printed pages and I blank page.

Copyright Zimbabwe School Examinations Council, 2000.

©ZIMSEC 2000. [Turn over

103
--~--------~--------------------------------------~--- --
Download more resources like this on ECOLEBOOKS.COM
2

NEITHER MATHEMATICAL TABLES NOR SLIDE RULES NOR For


CALCULATORS MAY BE USED' IN THIS PAPER Examiner'.
Use
1 Express.

(a) 2,5 mm as a fraction of 50 em in its lowest terms,

(b) 0,84 as apereentage,

2'
()C 14
,m mem. 2

Answer (a)
,
[1]

(b) _ % [1]

(c) em' [1]

2 Evaluate

~ 2/3
(a) ) X 51/3,

. (b) 163/4 ,

(c) (-U'

Answer/a) [1'

(b) [1]

(c) [1]

4008/1/4028/1 J2000

·104
Download more resources like this on ECOLEBOOKS.COM
3
For
3 Given that J7= 2,646 and 170 = 8 ~67 , calculate Examiners
Use

(a) -hooo,
(b) ~0,07.

Answer (a) [1]

(b) : [2]

4 Expand and simplify

(a) (x -3);

(b) 5(4x -7) -6(3x-2).

Answer (a) ......................•.... D]

(b) : ~ [2]

4008/1/4028/1 J200
[Turn over

'-' -;_;;",.

105 .
Download more resources like 4this on ECOLEBOOKS.COM

5 (a) Given that the bearing of B i"om A is 138", write down the hearing of A Fer
Kom B, expressing your answer as a three-figure bearing. frIIFllil •••
U.
(b) 'Theroad from A to B is represented 011 a map, draWlil to a scale of 1:50 000,
by a liDe of length 12 em.

Find the length, in kilometres. of the road.

Answer (a) fl]

(b) , .100[2]

<a> E~ 2s + 24 + 22 + 1 as a llumber in base 2.

(b) Given that 31. =16)0, for some base n,

(i) form an equation in n,

(ii) find the value of n.

Answer (a) 111 .


(b) (i) ..,'..... I.il

(ii) n = , : !: i
-------·-~-i
.&008/1/4028/1 J2000
I
I
I

106
Download more resources like this on ECOLEBOOKS.COM
s
7 ' Mary drew the pie chart shown to illustrate the time she speDt on her OOmewOJkill
'oJleweek. .

<a) Calculate the value of x.

(b) She spent 75 mmutes doing Science.

Calculate the total time, in hours. she spent doing her homework.

(c) Find the percentage of the time she spent on English.

Answer (a)" [1]

(h) h [IJ

(c:) ...................••.• % [IJ


_ _- - .. - .. -.- ._-_ .. _---_._ __ __ ---_. __ ._-- --_._. __ ._. __ .. '_', "--'-_",- _.- _._-_ .. _

..

rfurn .Iver
I

107
Download more resources like
6 this on ECOLEBOOKS.COM
For
8. Solve the simultaneous equations Examiner's
Use
2x+ 3y + 5 = 0

3x - 2y =12

Answer x = .

y = [3]

. 9 ABCDEFGHI is a regular nonagon with centre O.

(a) State the order of rotational symmetry of the nonagon.

(b) Calculate

(i) the size of one of the interior angles of the nonagon,


A

(ii) BOF

Answ,er (a) : [1]

(b) (i) [1]

(ii) SbF [1]

4008/114028/1 J200

108
10 Evaluate
7
Download more resources like this on ECOLEBOOKS.COM
.~.
for

u..
(a) .log., 16 -+- log102 , .

(b) 2 log105 + loglOJ6 - log!09.

Answer (a} .....••...........•........•.............


[ 1]
,..:._
-(b)••••
;•.•~••..••.•..••.•••.•••.•.•.••.•.....
[2]

4e<l8/11402811 J280

[Turnover

109
Download more resources like this on ECOLEBOOKS.COM
8

11 There are two types of factory machines available for sale. Fer
E ••••••
Their required floor spaces and prices are shown in the table below. ••••
, Type of Floor spa€e Deeded Price per
maemne per maehime (ml) maebme(S}'

A J 20000

B 2 45000

A fadory manager decides to buy x machifles of type A aad Y machines of type D,


wberex ~ 0 andy ~ O.

(a) If d:ae is 4t) m2 of floor space available, write down an inequality in x


andy.

(b) If the ~anager bas S400 000 to speDd., write down, in its si1nplest form.
aoother inequality in x and y. ,

I
I
·1

I
I

Answerta) r I]

(h) !ll

,
i
i
I
. , .
I
I
"

110
Download more resources like this on ECOLEBOOKS.COM

FIN
12 A bag contains (; red balls and 9 green balls which are all identical except for Examiner's
Colour. IJse

(a) Find, as a fraction in its lowest terms, the probability that a ball. takenat
random from the bag, is red.

(It) The ball is returned into the bag.

Two balls are then taken at random, one after the other without
replacement, from the bag.

Find the probability that they are of different colours ..

Answer (a) [1I

(b) [2]-:--

4008/1/4028/1 J200

[Turn over

111
--~~=',-, '
Download more resources like this on ECOLEBOOKS.COM
10
. .. i 2 .
13 (a) GIven that x + y = 12 and x .~y = 3O,evaluatex-y For
Examiner's
. Use
(b) Solve the equation

(t - 6t= 25.

Answer (a) x - y ~ ~ [1]

(b)t = :.or [2]

14 A tourist from England visited Zimbabwe bringing with him£ 2000(two thousand
pounds) which he exchanged for Zimbabwean dollars at the rate of £ 1 to Z$30 ..

(a) Calculate the amount he received in Zimbabwean dollars.

(b), After visiting various resort centres in Zimbabwe he had 'spent


Z$40 500.

He then exchanged the remainder of his money for pounds .

Find how many pounds he received.

Answer (a) Z$ ~ [1]

(b) £ [2]

4008/1/4028/1 J200

110
Download more resources like
11 this on ECOLEBOOKS.COM
For
Examiner's
Use
15 Tafadzwa left Mutare at 1.lOp ill by car arid arrived in Masvingo 3l hours later
He rested for ian hour and then travelled to Beit Bridge in 3 r hours.

(a) Find the time, on the 24 hour clock, when Tafadzwa arrived in BeitBridge.

(b) His average speed for the whole journey was 80kmlh.

Find the distance from Mutare to Beit Bridge

Answer (a) ~ [1]

(b) km [2]

16 (a) Express 5,9964 correct to 2 decimal places.


5
(b) Simplify (3,5 x 10 ).. (7 xl 02), giving the answer in standard form ..

Answer (a) [1]

(b) [2]

4008/1/4028/1 J200
[Turn over

113
Download more resources like this on ECOLEBOOKS.COM
12
For
17 (a) Divide 3 I by 5 t, giving the answer as a fraction in its lowest terms. EuminIr',
UIe
(b) Simplify 5 + 10 as far as possible.
~ 5 x 10

Answer (a) [1]

(hj ~............. 12]


.. _----- -------- .----1

3 . 1e fr action.
2 as a smg .
18 (a) . E xpress -- - ---
x-2 x

Y 23
(b) Given .that the matrix I/ + . )' has determinant 4, find the value ofy.
I-,y - 1 4. . .

An.nr(!r (a) · 121

(b)y= 12j
------- ------ - _--- --_._ -----

4HIII!/I!402l\/1 ,12OCHI

114
'.
Download more resources like this on ECOLEBOOKS.COM
13

19 (a) Solve the inequality 14 ~ 2 - 3x. Iex.m:..


lilt
'(b) The length, / em, of a side of a square is given as 6 em, correct to I
significant figure.

(i) Write down the upper bound of t.

·(ii) The area of the square is A em'.

Calculate the least possible value of A.

/ .

Answer (a) [l]

(b) (i) [l]

(ii) , [2]

4008/1/4028/1 J2000

[Thrnover

115
Download more resources like this on ECOLEBOOKS.COM
14
For
20. In this question take 7t tobe ~2. Examiner's
Use
A circular cylinder has hei ght 21 em.
Its volume is 1650 em'.
Calculate

(a) the radius of the cylinder,

(b) the volume 'of a similar body otheight 7cm.

Answer (a) .cm [2]

(b) cni [2] .

4008/1/4028/1 J2000

116
Download more resources like
15 this on ECOLEBOOKS.COM
For
Examiner's
21 Use
A

In the diagram. /1, lJ and ( are points on the circumference of a circle, centre 0:
TA and TB are tangents to .«, l'trc:k ACB = 56 and CBO =48
Calculate

(a) AOB,
(b) ABT.
(c) cA T.

AOB 1) [11
Ai31' " !) III.
(c) t-"'A' '1-- , .. , .... ,.,. 0 t. 121
-----'-------------- ------------ -- -- .. -.-.--.---------------- .. -- .----------------- ..
400S!V4tJi!!/I .I2!100

[Turn over

117
Download more resources 16like this on ECOLEBOOKS.COM

22 (a) Given that q -'- ~=r t -3 and i = 0. evaluate qrt'. For


Examiner',
Use
(b)Th(~ side of an equilateral triangle is 12 em. Find the perpendicular height
of the triangle in the fOITn m .r;;, where In and n are integers and n is as
small as possible.

Answer (a) [1]

(b) em [J]
..._ ...__ ._-----_ .. _--- ------------------ -- _-_._---_ .. _.
.. - ----- -------

I
I
1
j
I
I

I
I

I
I
i

118
Download more resources like this
17 on ECOLEBOOKS.COM

23

In the diagram, AFGD is a straight line.


AE is parallel to ED and EF is parallel to GB.
EF = 6 em, AF == FG = GD = 5 em and EFD = 64°.
(a) Name, in correct order, two triangleswhich are congruent.

(b) (i) Using as much of the information given below as is necessary,


calculate the area of triangle AGE. ~

(ii) Hence, or otherwise, find the area of triangle ABD.

[tan 64° ~ 2.u5; sin 64° = 0,90; cos 64" = O.44.J

Answer (a) [I]

(h) (i) ~AGn = em' [2)

(ii) /\ ABD ~-:: .cnr' p j


._ ._ ._----_. __ .._--_._--------_. _-_-_.- - .. _ .. _-_ .._--_--- -.. _ :'" _._---- -_
.. __ -- .. -_ .. _---- _.-_. __ .

119
Download more resources like this on ECOLEBOOKS.COM
18

24

Pattern 1 2 3 4

Identical sticks are used to make a sequence of triangular patterns.'


The diagram shows the first four patterns in the sequence.

Pattern Number of sticks Number of small Number of sticks .


Number in pattern triangles formed inside the pattern

1 3 1 0

2 9 4 3

1 18 9 9

4 30 16 18

5 x y z
-

The table above shows the number of sticks needed to form the patterns in the
sequence, the number of triangles in each pattern and the number of sticks that lie
. inside the outer boundary of each pattern.

(a) By considering the number patte~s in the table, write down the value of

(i) x,
_ (ii) y,
(iii) z.

400811/4018/. 12000

120
Download more resources like this on ECOLEBOOKS.COM
19
For
(b) Find the number of sticks needed to make pattern 10 in the sequence, . e.niIWl
U.

.;'_" ... - _"-:

Answer (a)(i) x = [1]


f'
(ii) y = [1]

(IIi) Z = [1]

(b) :.......... [1]

"00II11402lIl J2010

[Turn over

121
Download more resources like this on ECOLEBOOKS.COM
20
" .r
25 Three companies, Comfort Cars, Reliable Services and LUxury Cruisers, offer cars For
for hire. Their charges are based either on the number ot days for which a car is
hired, or on the number of kilometres for which the caris driven, or both, as shown
u..
EXlnilllrl

in the table below.

Company Cost per day Cost p~r kilometre


-
Comfort Cars $660 . Nil
--
Reliable Nil $4.50
Services

. Luxury $300 $2,50


Cruisers

(a) Mrs Tatenda hired a car for 2 days to drive 400km.

Find the cost if she hired the car from Luxury Cruisers.

(b) MrKudzai wishes to hire a car for 3 days.


He finds that Comfort Cars and Reliable Services would make equal
charges.

Find the distance he intends to drive.

Answer (a) $ " f2]

(h) km I:; t

4008/114028/1 .120110

122 ;~
; ,.
Download more resources like this on ECOLEBOOKS.COM
21
y~

26 I EDn:..·.
, ,- u.
4
\ A

\ 2

J
-8 -6 -4' -2/ 0 2 4 6 8 x

VB/ -2
I/
/ 1/ -4
V/ \
-6
'\
C 1\
. -8
\ f'

--'-- - ",_
-L. ___
.
The diagram shows quadnlaterals A. Band C.

(a) Quadrilateral A is mapped onto quadrilateral D by a reflection in the


line x::: J.

On the diagram draw and label quadrilateral D.

(b) A shear maps quadrilateral A onto quadrilateral B.

Describe fully this sne.ir.

(c) Describe tully the sinelc transformation which maps quadrilateral A onto
quadrilateral C.

Answer (a) On diagram [1]

tb) · .

........................................... 121

(c) ·;···
1-'

............ ' , .. - . I

•UIIIIII:"II:!X'I::!IlII!l

[Turn over

123
Download more resources like this on ECOLEBOOKS.COM
22

27

Velocity
(m/s)

Time (s)

The diagram shows the velocity-time graph of a car during a period of 14 seconds.
The car starts from rest and accelerates uniformly until it attains a velocity of 20mls
in 4 seconds,
It then retards uniformly to 8 mls in 2 seconds.
The car then moves at this velocity for a further 6 seconds and finaIly retards
uniformly until it comes to rest 2 seconds later.
Calculate

(a) the acceleration of the carduring the first 4 seconds,

(b) the velocity of the car 'at the end of the 13th second,

(c) thedistance travelled by the car during the last 7 seconds.

Answer (a) m/s' [lj

(b) mls [2J

(c) ....•................ m [2J

4008/114028/1 J2000

124
Download more resources like this on ECOLEBOOKS.COM
23
28 ;, . Two Points, A (5,2) and B (-3,8), lie on a straight line 1and C (-2, 4), lies on far
ExImIner'1
. 'another straight line m.
Use

(a) Find

(i) the gradient of 1,

(ii) AB as a column vector,

(iii) the length of the line AB.

, (b) Given that m is parallel to 1, find the equation of m in its simplest form.

Answer (a) (i) [1]

(ii) AB = [I]

(111) [2]

(b) [2]

.•

4008/1/4028/1 J2000
[Turn over

125
----~
Download more resources like this on ECOLEBOOKS.COM
24

BLANK PAGE

.•

126
Download more resources like this on ECOLEBOOKS.COM
MATHS JUNE '2000

4008/1

POSSIBLE ANSWERS

1
1. (a) - (b) 84 (c) 14000
200

2. (a) 5 (b) 8 (c) 9


3. (a) 83,67 (b) 0,2646

4. (a) x2-6x+9 (b) 2x-23 -

5. (a) 3180 (b) 6

6. (a) 11010h (b)(0 3n+1=16 (!i) 5


or110 101two

7. (a) 40 (b) 9 (c) 33! 0(33,3


3

8. (a) x=2 and y=-3

9. (a) 9 or nine (b)(i) 1400 (c) 1600


10. (a) 4 (b) 2

11. (a) 3x+2y:$; 40 (b) 4x+9y:$; 80

2 18
12. (a) - (b) -
5 35 , I

13. (a) 2% or2,5 (b) 1 or 11


14. (a) 60000 (b) 650
15. (a) 2025 (b) 580

16. ' (a) 6,00 (b) 5X102

2 3
17. (a) - (b) -
3 10

..

" ... .. 117


Download more resources like this on ECOLEBOOKS.COM

~.

TIME 2 hours 30 minutes

INSTRUCTIONS TO CANDIDATES

Write your name, Centre number and candidate number in the spaces provided on the answer paper/answei
booklet.
Answer all questions in Section A and any three questions from Section B.
Write your answers on the separate answer paper provided.
If you use more than one sheet of paper. fasten the sheets together.
All working must be clearly shown, It should be done on the same sheet as the rest of the answer.
Omission of essential working will result in loss or marks,
If the degree of accuracy is not specified in the question and if the answer is not exact, the answer should be
given to three significant figures. Answers in degrees should be given to one decimal place.

INFORMATION FOR CANIHOATES

The number of marks is given III brackets I I at the end of each question or part question.
Mathematical tables or electronic calculators may be used to evaluate explicit numerical expressions .

.•
---"':----------' _,_,--,--,,-'-----,,-----, ----
This question papa consists of 12 printl'd pages.

Copyright Zimbabwe School lxuminations t ,'wlt'iI


"'ZIMSEC ~OOO (Turn over

129
Download more resources like2 this on ECOLEBOOKS.COM
Section A [64 marks]

Answer all the questions in this section. -

(a)
a+b
G·rven th at y= --.
'b .

(i) calculate the value of y when a = 40 and b = 0,1, [2]

(ii) express b in terms of'j-and a. [3]

(b) Given thatj(x) = 2 x? + 7x, calculate

(i) j(3),

(ii)the values of x for whichf(x) = 4. [4]

(i) P+3Q, [2]

(ii) - [2]

("')
In the inverse of P. [3]

(b) ~= { x: 10 -c x ~ 31} where x is an integer.

A = {x: x is prime}.

B = {x: x is even}.

C = {x : x is a multiple of 5}.

(i) List all the elements of set A. [IJ

(il) Find (a) n(C), [1]

(b) nCBu C). (2J

(iii) Express in set notation, using some or all of A, Band C,


the set {lS, 25}.' {2]

402!;/} _i 10f)()

130
Download more resources like this
3 on ECOLEBOOKS.COM

3 (a)'. Factorise completely

(i) ~.• '

(ii) as + 2at- 3s - 6t; [4]

(b) If y varies directly as the cube of x and y = 40 when x = 2, find the value
ofy when x = 3. [3]
__________________________________ ~'Jo._A_....._
4

12

In the diagram, ABC is an acute angled triangle in which AB = 12 em, BC = 15 em


and cAB = 58°,
(a) Calculate

(i) ACE,. [3]

. (ii) the perpendicular distance from C to AB, [3]

(iii) ·.1C [2]

(b) Given that L lies on AC and that AL = 6 cm, calculate BL. [4]

4028/2 J 2000
(Turn over

131
Download more resources like this on ECOLEBOOKS.COM
4
.5 A salesman was paid his annual salary iri twelve equal monthly amounts. In addition at the
end of each year, he was paid a bonus which amounted to 31
% of the value of his total annual
s~es.

Given that his annual salary was $39 000 and that his total sales during the first year
amounted to $100 000, calculate

(a) his monthly salary, . [1]

(b) his total income in the first year. [2]

During the second year his annual salary remained unchanged hut his total income
for the year amounted to $44 145.

(c) Calculate his total sales during the second year. [4]

In the third year his annual salary was increased to $46 800 and his bonus was
also increased.

(d) Calculate

(i) the percentage increase. in his annual salary, [2]

(il) his new percentage honus payment, given that his total income
during the year was $50,000 and his sales totalled $80 000. [3]

132
Download more resources like this
5 on ECOLEBOOKS.COM

6 Ar.swer the whele of this question on a sheet of plain paper.

Use ruler and compasses only for all constructions and show clearly all the construction arcs
and lines in a single diagram.

A farmer has a field in the shape of a quadrilateral ABCD.


The field is bounded by two straight roads, AB and BC with ABC = 90°, and two straight
fences CD and AD with BCD = 120°. '
It is given that AB = 70 m, BC = 90 m and CD = 50 m.

. (8) (i) Using a scale of 1 em to represent 10m, construct an accurate scale


drawing of the field. [4]

(ii) Measure and write down BAn. [1]

(b) The farmer has a well in the field which is equidistant from AB and BC and also
equidistant from A and B. '

Construct

(i) the locus of points which are equidistant from AB and BC. [2]

(U) the locus of points which are equidistant from A and B, [2]

(c) 0) Mark and clearly label W, the position of the well in the field. [I]

(ii) Find the distance of the well from the comer D of the field. III

!TIIt'n un'l"

133
Download more resources like this on ECOLEBOOKS.COM
6
Section B [36 marks]

Answer three questions in this section.

7 A car travels from Bulawayo to Harare by a route 440 km long, at an average speed
ofxkm!h.

(a) Write down an expression, in terms of x, for the time, in hours, needed
for the journey [1]

(b) A mini-bus travels from Bulawayo to Harare by a different route, which


is 8 km longer.

The average soeed.of the mini-bus is 4 km!h less than that of the car.

Write down an expression, in terms of x, for

(i) the average speed, in kmIh, of the mini-bus, [1]

,(ii) the time, in hours, needed for the journey by the mini-bus. [1]

(c) It is given that the mini-bus takes t an hour longer than the car to travel
from Bulawayo to Harare.

(i) Write down an equation which x must satisfy and show that it
simplifies to

x2,- 20x - 3520 = O. [3]

(ii) Solve this equation, giving the answers correct to 3 significant figures.

Hence find the average speed of the mini-bus, giving your answer
correct to 3 significant figures. [6]

402812 J 2000

134
r-
Download more resources like this on ECOLEBOOKS.COM
7
-
8

A
B

In this question take 1t. to be 3,142.

In the diagram AOB is a sector of a circle, centre O.


The radius of the circle = 6,5 em andA6B = 120°.

(a) Calculate

(i) . the length of the arc AB. [2]

(ii) the perimeter of the sector AOB. [1]

(iii) the area of the sector AOB. [2]

(b) If this sector is the cross-section of'a solid of length 12 em, calculate

(i) the volume of the solid, [2]

(ii) the density of the solid, given thatits mass is 4192 g, [2]

(iii) the total surface area of the solid. [3]

402812 J 2000 .
(Turnover

135
Download more resources like this
8 on ECOLEBOOKS.COM

9 Answer the whole of this.question on a sheet of graph paper.

Below is an incomplete table of values for the function

Y= l.r-2x+3
2 ,

for values -1 :5; x :5; 6.

x -1 0 1 2 3 4 5 6

y a 3 1,5 1 1,5 3 5,5 b

(8) Calculate the values of a and b. [2]

(b). Using a scale of2 em to represent 1 unit on each axis, draw axes for values
ofx andy in the range -1 Sx:5; 6 and O:5;y S 10.

Plot the points represented by the values in the table and draw the graph
of y = t r -2x+ 3. [4]

(c) Use your graph to estimate

(i) the gradient of the curve when x = 3, [2]


~
(il) the area of the region between the curve, the x-axis and the
lines x = 0 and x = 4. [2]

(d) Hence; or otherwise, find the area of the regionenc1osed by the curve
and the line y ~ 3. [2]

4028fl J 2000

136
---------;

Download more resources like this on ECOLEBOOKS.COM


9
. ,

10 'Answer the whole of this question on a sheet of graph paper.

A fanner had 100 pigs. He weighed them and their masses were recorded and
summarised as shown in Table 1 beJow.

Table 1
,

Mass 40 <m:5:60 60 < m :5:70 70 < m S 80 80<ms90 90<m:5:100 100 <m s 140 ,
(kg) , !
Number I
I
I
!
\
of pigs 12 14 1 1·8 22 14 20

Table 2 is part of a cumulative frequency table for the distribution.

Table 2

Mass mS40 mS60 m~;70 m:5:80 m:5:90 m s; 100 in S 140 . /


(kg)
Number -
of pigs 0 12 26 P 66 q 100

(a) Calculate the values of p and q. [2]

(b) Using a horizontal scale of2 em to represent 20 kg and a vertical scale of


2cm to represent 20 pigs, draw a smooth cumulative frequency curve to
illustrate this information. "
[4]

(c)' Showing your method clearly on the graph, use your graph to estimate

,(i) the median mass,

(ii) the number of pigs whose masses were more than 65 kg but less
than 120 'kg. [4]

4028/2 J 2000
{Tum over

137
Download more resources like this on ECOLEBOOKS.COM
10
(d) J:he data in Table 1 may also be expressed in the form given in Table 3 below.

Table 3

Mass (kg) 40 <m:S;60 60 < m s; 80 80 < m s; 100 100 < m s 140


.Number
;
of pigs 12 32 36 20

When a histogram is drawn to illustrate this information, the height of the column
representing pigs with mass WI in the interval 40 < m :s;60 is 2,4 em, .
Without drawing the histogram calculate the height of the column that represents
values of m in the ranges

(i) 60 < m s 80,

(ii) l00<m~140. [2]

402812 J 2000

138
Download more resources like this on ECOLEBOOKS.COM
II

11 Answer the whole of this question on a sheet of graph paper.

Triangle A has vertices at (1,2), (2, 2) and (1,4).


Triangle B has vertices at (-2, -1), (-4, -1) and (-2, -5).
Using a scale of2 cm ro represent 2 units on each axis, draw axes for valuesofx andy in the
ranges -6 s x s 8 and -10 sY s 8.

(a) Draw and label triangles A and B. [2]

(b) Triangle B is the image of triangle A under a certain single transformation.


Describe fully this transformation. [3]

(c) Triangle C is the image of triangle A under a two-way stret7h in which\the o~gin
is invariant. The stretch has scale factor 3 parallel to the x-axis and scale factor -2
parallel to the) -axis,

Draw and label the triangle C. [3]

0 -I') .
(d) The transformation X is represented by the matrix (. I 0) and maps triangle A onto'

.rrianzlc
~ 1).

(i) Find the coordinates of the vertices of [)

Draw and label the triangle D.

(ii) Describe fully ! he single transformation X. f41

[Tur» u\er

139
Download more resources like this on ECOLEBOOKS.COM
12
11

'.
In the diagram, OA = a, OB .= b.
The point C is such that AC = 3CB and the point D is such that-Of) = DA.

(a)· Express, in terms of a and/or b, the vectors

(i) AB,

(ii) OD,

(iii) AC,

(iv) ~C. [4]


_._
(b) DC and BD meet at X Given that BX =·k BD , express BX in terms of 8, b

and k. Hence show that OX =


- t ka + (1 - k) b. [3] .

(c) Given also tnat OX = hOC, express OX in terms of a, b and h. [I]

(d) Using these two expressions for OX, find the values of hand k. [3]

(e) Find the numerical value of the ratio BX : XD. [1)

402812 J 2000

140
Download more resources like this on ECOLEBOOKS.COM
MATHS JUNE 2000
4OO8u2

POSSIBLE ANSWERS

. f (8) (i) y= 40+0.1


0,1

OR 40,1
0.1 M1

=401 A1
(ii) yb=a+b M1
b (y-1}=a M1
b=_!!_.
Y-l A1

(b) (i) 1(3)=2(32)+7x3


=39 81
(ii) 2X2 + 7X =4 M1
. -7±.J8i
(2x.,.. l)(x + 4) = Oor x= 4· M1

x~ 15or-4 A1
[9]

2. (a) (i) P+3Q=( -4I


-7}3(2
3
-3)
4--1

2(:3' -1:) 82

(ii)Q"=C -3W-3)
. 4-1 4:-1

;,(-8 -3 J
4:....11 82

. ! {. 141
Download more resources like this on ECOLEBOOKS.COM

(iii) det P=-12+7=-5 ·81

P=- -l( 3 7)
5 -1-4
83

(b) (i) A={11 ;13;17;19;23;29;31} 81


B={12;14;16;--- ----;18;30}

(ii) (a)' C={15;20;25;30}


n(C)=4 ' 81
(b) n(Bu C)=12 . 81

(iii) {15;25}=B' r'I C 82


OR Cn(BuA)' OR A' n .B' r'lC
[13]

3. . (a> (i) 8p3 - 18p = 2p(4p2 - 9) M1


. =2p(2p-3)(2p+3) A1

.'. (ii) as+2at-3s-6t=a(s+2t)-3(s+2t)


OR s(a-3)+2t(8-3) MAt

(a-3)(s+2t) A1 .

M1

A1

y=5(3~

=135 A1

[7]

142
Download more resources like this on ECOLEBOOKS.COM
"
SinACB Sin58°
4.(a) (i) .
'\
12 15

Sin ACB= 12Sin58° M2


15

A1

(ii) ABC =79,3° OR 79°17' B1


!!__ =Sin (180°-58°-42 7f)
15 '

Perpendicular =15 Sin (180°-58°-42,7°) M1

=14,74cm A1

(iii) 14,74
-. -_ _ S'In 580
AC

AC= 14,74 M1
Sin58°

=17,38cm A1

(b) BL2=62+122_2x6x12 Cos 58° M1

=103,69 A1

BL= .JI03,69 . M1

=10,18cm A1

[12]

. 143
Download more resources like this on ECOLEBOOKS.COM

39000
5. (a) Monthly salary =
12
81
=$3250
.' .
M1
(b) . Total income = 39000+3~%x100 000

$42500 A1
=

(c) 44145-39000
81
=5145

100 . "'M2
Total sales =-x5145
. 3.5

A1
=$147000

0A . . I 46800 .; 39000 ()() M1


(d)(i) o Increase In sa ary = xl
39000

=20% A1
(ii) 50 000-46 800
81
=3200

3200 M1
% bonus = xl 00
80000
A1
= 4%
[12]

6.

144
Download more resources like this on ECOLEBOOKS.COM
(a) (i) ABC =900 ± 20 with correct construction arcs' seen 81
A CD = 1200 ± 2° with correct construction arcs seen 81

Quadrilateral with sides 9 ± 0,2cm, 5 ± 0,2cm,


7 ± O,2cm and ABC =900± 2° and 8CD=120o± 2° Q2

(ii) BAD =77°± 2° stated or labelled B1

(b) (i) Bisector of ABC ruled, at least 3cm long with


tolerances ± 2° 81

Correct construction arcs seen 81

(iii) Perpendicular bisector of AS ruled at least 3cm long 81

Correct construction arcs seen 81

(c) (i) W clearly labelled at the intersection of the angle


bisector and the perpendicular bisector W1

(ii) WD=81 ± 2m 81

[11]

440 L_
()a T nne=-rlf'S 81
o

7.
X

(b) (i) Average speed =( x - 4 )kmlhr 81


.. ) TOline ~ror rmm-b'us =--.
( II ° ° 448 hrs 61
0", x-4

( c) (0)I •-.---=1'2
448 440 v M1
x-4 X

2x( 448) - 2( 440)(x - 4) = x(x - 4)


896x - 880x + 3520 = Xl - 4x
16x+3520=x2 -4x
Xl -20x-3520 = o (with no wrong working) A2

145
Download more resources like this on ECOLEBOOKS.COM

-b±.Jb2 -4ac
. (ii) x=-----
2a
:: 20±~(20)2 -4xlx(3520)
2

20±.J14480
=---- 8181
2
81
=70,2 or- 50,2 .8181

Alternatively by completing the square,


(x _10)2 = 3520+100 81

(x - 10) = .J3620 81

x=10± .J3620 81

~=70,2 or 50,2 B1B1

Speed mini-bus = x-4


=66,2kmlh 81 . [12]

8. (a) (i) length of arc = 120 x21Z'x6,5 M1


360
=13,61cm A1

(ii) Perimeter of sector=13,61 +6,5+6,5 M1


=26,61 em A1

(iii) Area of sector- 120X1Z'X


(6,5)2 M1
, 360
=44 ,24cm2.. . A1

(b) (i) Volume of solid=44.24x12 M1


. =530.9cm3 A1

(II")0ens I'ty 0f SOl


I'd=--4192 M1
. 530,9
- . =7,895 g/cm3 A1

146
Download more resources like this on ECOLEBOOKS.COM
(III) Total surface area
=2x6,5x12+2x44,24+13,61x12 M1

=407,76 crrt A1·


[12]

9. (a) a=5,5 81
I)=~ '8t.

(b) Refer to graph. The 6 given points and (-1;5,5)


And (6;9) correctly plotted P3
Smooth curve, not grossly thick, passin through
all 8 correct points . C1

(c) (i) Tangent drawn to the curve at x=3 M1


Gradient =1,00 /\1

(ii) Area =6,67±0,16 (units)2


Or 26,7 ± 1,60 cm2 (after converting using the s.. 82

(d) Area = 12-6,67 l' M1


= 5,33 ± 0.,41 (units)2
or 21,39± 1,63 cni2 A1
[12]

147
Download more resources like this on ECOLEBOOKS.COM

148
Download more resources like this on ECOLEBOOKS.COM

10. (a) p=44 81


q=80 81

Refer to graph.
(b) The 5 given points and (80;44) and (100;80)
Correctly plotted P3
Smooth curve, not grossly thick passing through
The 7 correct points. C1

(c )(i) Dotted line (or marks) drawn at y=50,5 M1


Median=83 A1

(ii) Vertical lines (dotted) drawn at )(:;65 and x=120,


Or marks seen on graph M1

No. of pigs=96-18
=74 .
A1

12
(d) (i) x x=2,4
60-40
:. x= 4

- 32
For 60 < m S; 80, height =- x4
20
=6,4cm 81
For 100 < m S; 140, height = 20 x4
40

=2cm 81
(12)

149
-----------------------------~~~~~~~-------~~-
Download more resources like this on ECOLEBOOKS.COM

, 1-+-' , I

- -=+. f~=tm-=i=i
i.,,;!

+
,. , !
I

tt.~ :t__
,'-_:+.j,
LL:::t _
fiT ~'+:,m, I +- '
+-+-,-r+++-+-i-+-+-i-+--'-.,' --;-+--j,-+-j+,_;_-i-t-,
:...:r++'
, t~- r+: '-+1 ~, . .
ft _ ~ ! l~;+=4".
·-tT .
i±T~III"
.' ' -;~.
: '~+~: t-EEL_:_:-;-'-i-H-i-H--i-l-+-;-+-+'-r--H-i-+-+-i-ri-+H-+-+-+++-++H-i-+-+-i-+-+
'p,1! ,~_~
~ H _
; r
+-'
~J_'
I 1 I, 1
,I.
_,;'
' 1'..',
j-, '-i-. '-i-, rt+rrrt:
.t--~'-t--,t--t--H-H-i-+-t--t--H-t--t--H-H-f-'-t--t--t--t--H-iH-+
±
.u+-, '_~, _'_'.'

'-;-
-4-- ;
1-,-+
Ie ' '-+'++--
!+l :~----: '+--+-+-++-1-++ -'-4
I rr
. ' -t-t--i-ri--rl-t-t--i-..;....,.-+-t-l-+++r_;,._;_il--'-+--+-++-H-++' III i
! I
rr+r+t+t+t+t
r r r t t t r i i :
•+l+l-l+l+l+i+l-l+l+ r-+++H- i+i-' "f,-+-:-
+1+-
_J-!-L:
~,
,, l+ +, I·
+-' , :

I_
p_.

+++-, ;
" ; +t
'liI'._'

. ,--,--;-H-'--I-t-+-'i-"-+'-+, +-: i, l+t+


---4-1 ! :-~ '_t~~
++,--i"'1'--i--l-+-H--+-H-+~.4, -T-"" +-'-: + ,--r- --j I ,- .

H-" '---T-~H-++-H-++
t+m~=+'
; t [ ~j. i -

::;:H-:+' ' l'+T-tf-+t;:: ,~±±tt- T-'-':_;_ ___::::::...:_,'H:-,-+~-r+·+++-t--t--H---t--T-Hlicft+++---ri-H--++++-rl -t++++++-I-H-+-t+t-+++t--+--t--i-rl-+--t-+++++-++


-t+~i117 ,I-+tl++f- ~H--;---, _. ~+'- -H-+-H+-H-+-++-iVCl~-#-+-l +'-i--H-i--H-+-t-t-+-i-+-+-i--H-++-+-+-i-H-i--H'-t-i-+-+-i--H-l--+-H-i-+-+-+-H-t-

! 1

150
Download more resources like this on ECOLEBOOKS.COM

11. Refer to graph


(8; . Triangle A correctly drawn T1
Triangle B correctly drawn E1
(b) Enlargement
M1
Centre (0;1)
A1
Scale factor -2
A1

(c) Triangle C correctly drawn with vertices at (3;-4),


(6;-4) and (3;8) S3

3
from (
lo -2OJ(1. 2 2
2 4
1J = ( -43 -':46 -83J

(d) (i) (0 -lJ'(1 21J=(


10224
-2 -2 -4J
\1 21
Coordinates are (-2;1), (-2;2) and (-4;1) 81
Triangle D correctly drawn . T1

(ii) Rotation 1M1


. Centre origin or (0;0) and through 90 anti-clockwise
Or 2700 clockwise A1
[12]

9
Download more resources like this on ECOLEBOOKS.COM

+r+

l.•. ~_
• . j . ~".. -~...;.--, .; .

t. ':.:
._I
'o"o'+j: :....

...J.,
.- d-1t-: ~:~
,~J~." .-

152
Download more resources like this on ECOLEBOOKS.COM
12 -- -
(a) (i) AB=AO+OB

=-a+b
81
(ii) OD=~a
81

(iii) AC =% AD =%( -a+b) 81

(iv) OC = OR +%BA
= b-% (-a+b)

= %a+%b
81

(b) BX =kBD

=k(~a-b)
81

OX
----.
=OB+BX - l'

=b+k(~a-b)
M1
=YJca+(1-k)b
A1
~
(c) ox . =h(Y..a+%b)
81

(d) YJJa+YJJb=%ka+(1-k)b
M1
YJJ=YJc :. h=2k
e'
%h=1-:-k

%~k=1-k

5
-k=1
2

2
k=-
5 A1

10

153
Download more resources like this on ECOLEBOOKS.COM

4 A1
h=-
5

(e)
- -
XD=(1-k)BD

=~(Y:za-b)
5 .

2 3
:. 8X:XD= - (Y:za-b) : - (Y:za-b)
. 5 5
81
=2:3
[121
A:EGl4008-2-JOO

11

154
Download more resources like this on ECOLEBOOKS.COM
Candidate Centre Candidate
Name Number Number

ZIMBABWE SCHOOL EXAMINATIONS COUNCil


General Certificate of Education Ordinary Level

MA THEMATICS 4008/1, 4028/1


PAPER 1

Wednesday 8 NOVEMBER 2000 Morning' 2 hours 30 minutes

Candidates answer on the question paper.


No additional materials are required. 159136'
TIME 2 hours 30 minutes

INSTRUCTIONS 'TO CANDIDATES

Write your name, Centre number and candidate number in' the spaces at the
top of this paqe, ~
Answer all questi ,.,ns.
Write your answers in the spaces provided on the question paper.
If 'working is needed for any question it must be shown in the space below
that question.
Omission of essential working will result in loss of marks.
Mathematical tables, slide rules and calcuiators may not be brought into the
examination room.

INFORMATION FOR CANDIDATES

The number of marks 'is given in brackets [ j at the end of each question or
part question.

FOR EXAMINER'S ust;:

This question paper consists of 22 printed pages and 2 blank pages.


1

Copyright: Zimbabwe Schoo,1 Examinations Council, 2000.

<e>ZIMSEC 2000 {Turn over

155
Download more resources like this
1,
on ECOLEBOOKS.COM

NEITHER MATHEMATICAL TABLES NOR SLIDE RULES NOR


CALCULATORS MAYBE USED IN THIS PArER

1 (a> Evaluate, giving your answer in decimal form, 1,S~9- O~On~1.

(b) Express as a fraction in its lowest terms,

(I)

(0) _-.S +...."....


7.
16 12

(a).................... [1]

(b) (i).••.•..•••....•..[1]

(ii)................ .[1]

2 Given that p :lIZ S,q =- -2 and r = 1, find the valueof


(a) pc(r,

(b) 13-2(q -p).


"

(a) .••.......•.•.••••.[1]

[.20]
(b) ...•...•••••••...•.

156
Download more resources like this on ECOLEBOOKS.COM
3

3 in terms of time, Japan is 7 hours ahead of Zimbabwe when Britain is 2 hours For
Examiners
behind Zimbabwe. Use

<a> Calculate the difference in time between Britain and Japan.

(b) , It is 1135 in Zimbabwe. Giving your answer as time on the 24 hour clock,
calculate the time in

(i) Britain,

(li) Japan.

Answer (a) .. ; - .. (lll I

(b) (iJ.-............... f1 J i,
(ii) . [1] 'Ii

--------------------------------------------------------------,
4 Ca> Calculate the highest common factor (H.C.F) of the three I
numbers 18,30 and 42.

(b) Three uniform rods have lengths of 1,8m, 3,Om and 4,2m., Kuda cuts pieces
of equal Iengths from each of the rods. State the greatest possible length of I
each piece. t

I
I
I
j
I

I
iI
I

I
!
Answer / i
,G, ' ;;I. ,j!
[._]

!
tb) [1);
-------------------.------'-'-'-.-::-----~
"IiO!!l 1402&11 ~"2COO

157
~- ------

Download more resources like 4 this on ECOLEBOOKS.COM


For
5 (a) In the diagram, shade the set (pnQ)UR. Examiner's
Use
p

[1]

(b)
A B

c
..

Use set notation to describe the shaded region in the diagram.

Answer [2]

400&1114028/1 mooo

158
Download more resources like this on ECOLEBOOKS.COM
s
6 The number of pupils at a school to the nearest 10 is 450. For
State the greatest possible number of pupils at the school. Examiner'.
u.e
(b) A magic square is an array of numbers with a special property that the sum
of all the numbers in a row, or column or along a diagonal is always the
same. This sum is called the magic number of the square. .

401 227 . 179

P 269 491

359 311 137

Using the magic square above, calculate

(i) the magic number of the square,

(ii) the value of P.


f'

Answer (a) ....•.................... [1]

(b) (i) [1]

(ii) .•. [1]

7 . 1:~·~ -4-2-
SlIDp.uA.) b
(a> •
abc
.
(e) Solve the-equation m2 + 7m - 18 = O.

Answer (a) [1]

(b) m = .. or [2]

400&111402811 NlOOO
[Turn over

159
Download more resources like
(; this on ECOLEBOOKS.COM
-
An:orange syrup was diluted with water in the ratio l:S respectively.
The volume of the orange syrup used was 3SOml. Calculate

<8> the volume of the diluted drink,


, \

(b) the number of people who can be served, if each is given exactly 80m! of
the diluted drink.

Answer (a) mll2.]

(b) [I]

9 A number in standard form is 6,714 x 1023.

<a> State the number of zeros in the number when it is written in


ordinary form.

(b) Express in standard form

(i) twice the number,

(ill one-millionth of the number.

[1]
(a). .•.•.••.••••••••.

(b) (i) ..•..... .... [1]

(ii) •............... [1]

4OOII1I4Q2IIl NlOOCI

160
Download more resources like this on ECOLEBOOKS.COM
7
10 (a) Convert 10012 to a number in base ten.
. ,

':".' ;,
(h) Subtract 325 from 415, giving your answer in base five.

(e) Convert 27 to a number in base five.

"

Ii

Answer (a) [1] \

(b) •••..•.•.
.•.•..•.•••••
••••[1)

(c) ...........•...... ..... ... [1]

11· <a) Evaluate. (4 2.( S).


\~17

,.
Answer . Ca> ••••••••••••••••••••[1]
(1)>) ••••••••••••••••••••• ~]

flUnaover

161
Download more resources like this on ECOLEBOOKS.COM
8 '

12

The graph shows straight lines 11 and 12 intersecting at A. Q.is the origin. Line 11
cuts the x-axis at C, and line 12 cuts the x-axis at B.

Ca> Find the area of AABC.

(b> C is the image of A under a reflection in the line 13. Draw the line 13•

Answer (a) unitsl [2]

(b)_ongraph [IJ

13 Evaluate <a>
(b~ log.32 + lo~.

Answer (a) ... ~...•...........• [1] ,

(b) ••....•.•......•.•. [2]

.wo8l1l402&ll N2000

162
Download more resources like9 this on ECOLEBOOKS.COM
14 Given tbat-S ~x ~ 1 and 6 ~ys17,find

<a> the greatest value ofy-x,

(b) the least value of ~ - y.

Answer (aJ •..••••••••••••••


[1]

(b) ••.•....... ... [2]

..•..,

. ~. - .

4008111402111 NloOo

(Turaover

163
Download more resources like
10 this on ECOLEBOOKS.COM
Far
" A map is enlarged in the ratio 2: 1. The original scale was 1: 250 000. EDmNr'1
u.
(a) Calculate the new scale in the form 1: n.

(b) A lake has an area of 56cm2 OD the original map. Calculate the area of the
lake on the enlarged map.

Answer ra) 1:...... .... [1]

(b) cm2 [2]

4008I1I402811 N2000 '

164 ...
~.
Download more resources like
11 this on ECOLEBOOKS.COM
far
A till

In the diagram, ABC is a triangle and AD is perpendicular to BC produced .


. AC = 5cm, AD = 3cm, Be = 7cm and CD = 4cm.

(a) Find, as a common fraction

(i)

(ii)
.
tan ACB .

(b) Write down the ratio of the area of 6 ABC: the area of l:J.ACD in its
simplest form:

Arswcr {a] (0 sin ACE = -.... [1]·


.
(U) tanACB "" [I]

(h) area. (If ~.ABC: area of .6.ACD .

= . [1]
---------_._---_._----_._. __ __ ._--
.

4()O'V1/402&11 !'.':(i(l()

- [Turn over
165
Download more resources like
11
this on ECOLEBOOKS.COM

17

In the diagram, DA.= a and DB = b.

The points C and D are also shown on the diagram.


f'
<a> Mark and label clearly on the diagram the point X such that
-
DX =2a+3b.

(b) Write down DC in terms of a and b.

(e) Draw DE such that DE = DC.

Answer (a) on diagram [I]

(b) - [1]
OC - ••...•••••••

(e) . on diagram il]

. «J08II/402III N2000

166
Download more resources like13this on ECOLEBOOKS.COM
Far
ex.nInefa
18 . Use
s
T

P, Q,R and S are points on a circle, centre O. TS is the tangent to the circle and
is
TPQ a straight line that cuts the circle at P and Q. ST~isparallel to RP,
STP = 26° and SOR = 134°.

Calculate (a) SPR,

(b) asp,
(c) SPT.

Answer (aJ SPR = [1]

(b) OSP = ······Ul


(c) 'sir = [2]
, \

4008/110402111 mooo
[Turn over

167
Download more resources like this on ECOLEBOOKS.COM
14
. For
19 ExImJnIrI.
U.
BEITBRIDGE 255 BULAWAYO 66
GWANDA

The sign post shows the information displayed at a junction on the


Bulawayo-Beitbridge-road. ~

<a> Calculate the distance between Gwanda and Beitbrldge, given that
Gwanda is 126 Ian from Bulawayo. -

(b) Calculate the time taken by a motorist who travelledbetween


. Bulawayo and Beitbridge at an average speed of 107 kmIh.

fa) .•...•. km [2]

(b) ....•••.... h [2]

168
Download more resources like this on ECOLEBOOKS.COM
13
For
ExImIner".
18 . U••

s T

P, Q, R and S are points on a circle, centre o. TS is the tangc;:ntto the circle and
is
TPQ a straight line that cuts the circle at P and Q. Sri'is parallel to RP,
STP = 26° and SOR = 134°.

Calculate (a) "


SPR,

(b) asp,
(c) SPT.

Answer (a) SPR = [1]

(b) OSP = ···Ul


(c) spr = [2]
, \

4008l1l402S11 N2000

[Turnover
". ,,

167
Download more resources like this on ECOLEBOOKS.COM
14
. For
19 ExIminIr'I
U.
BEITBRIDGE 255 BULAWAYO 66
GWANDA

The sign post shows the information displayed at a junction on the


Bulawayo-Beitbrldge-roed. ~

(a) Calculate the distance between Gwanda and Beitbridge, given that
Gwanda is 126 Ian from Bulawayo.

(b) Calculate the time taken by a motorist who travelled between


, Bulawayo and Beitbridge at an average speed of 107 kmIh.

ra) .•.•.••Ian [2]

(b) ....••..... h [2]

168
Download more resources like this
15. on ECOLEBOOKS.COM

20 (a) ~ 1,Sglcm1inkg/ml.

(b) Given that f(x) = tal + Sx + 6,

calculatethe value ofk given that f(3) = -60.

Answer (a) ......•..... ~ {2)

(b) k= [2]

4OO&I1J402&11 N2000 .

[Turnover

169
Download more resources like this on ECOLEBOOKS.COM
16
For
21 (a) A plane is flying on a bearing of 293°. It then changes 'direction Examlner'1
through 83° anticlockwise. Calculate its new bearing .. Uu
(b)
c

In the diagram, AB and Be are adjacent sides of a square. AB and BX are


adjacent sides of a regular n-sided polygon.

Given that CM=- 126°, calculate


••
(i) reflex CBX ,

(h") the exteriorangle of the n - sided polygon,

em") the value of n.

Answer (a) •..•..••.•...•.•••• [1·]

(b) <') reflex CBx= [1]

(ii) ..............•. [1]

(iii) n =.............. [1]

400811/402811 NlOOO

170
Download more resources like 17this on ECOLEBOOKS.COM
For
, 22 (a) The shape shown is an isosceles trapezium. On the diagram draw the ,ExIminer'.
line of symmetry. ' Use

(b) The parallel sides of an isosceles trapezium are of lengths Scm and
18cm, and its perpendicular height is 12 cm. Calculate the length of
one of the non-parallel sides.

Answer (a) on the diagram [I]

(b) ........•......... CIIl [3]

4008/114028/1 N2000

[Turnover
171
Download more resources like this on ECOLEBOOKS.COM
18
.
Study-the pattern below.

Line! 2+3=5
-
Line2 2+6=8

Line3 2+9=11

- - -
- - -
- - -
Lin~n ,2+b=x. .

-
Use the pattern to

Ca> write down line6,

(b) write down the value ofb in terms of n, .

(e) show that xcannot be 42.

Answer (a) 1inc6 :.. ~... raJ·


(b) b =.................... ... [1].

(c)

......•.•....•..........•..•

...........
"\
.,........... [2].

. 172
Download more resources like this on ECOLEBOOKS.COM
19

24 In this question take 1t to be 22 •


7

x
Figure A Figure B

Figure A and Figure B have the same volume. Figure A is' a cone of base .
radius 9cm and height 28cm. Figure B is a prism whose length. is 22cm, and
whose cross-section is a right-angled isosceles triangle with the base and the
height each equal to x em.

Calculate (a) the volume of the cone,

(b) the value of x, leaving your ans~er in surd form.

[Volume of cone = tnih where r = base radius and h = height.]

-, ,

Answer (a) :.. cm) [2]

(b) x = [2]

4008111204811 N2000
(Turn over

173
Download more resources like this on ECOLEBOOKS.COM
20

2S A rectangular concrete slab is 4m long, 1t m wide and 20·cm thick.

(a) Calculate the total surface area of the slab, giving your answer in cnr'.
(b) The whole slab is to be painted at a cost of $15,00 per square metre.
Calculate the cost of painting the slab.

Answer (a) cm2 [2]

(b) $................. [2]

26 (a) A discount of 15% is given 011 all goods that are sold for cash in a
shop.

Calculate the cash price of an article that has a marked price of $68,00.

(b) The price of a magazine was $16,50 in 1999. Given that this was an
increase of 10% from the 1998 price, calculate the price of the
magazine in 1998.

Answer (a) $.................. [2]

(b) $ [2]

4008/114028/1 N2000

174
r
Download more resources like21 this on ECOLEBOOKS.COM
27

40

Speed
m/s

1()

t time In S

The'diagram shows the speed-time graph of a train which retards in two stages
until it comes to rest after a time of t seconds.

(a> Calculate

(i) the retardation of the train during the first 60 seconds,

(ii) the distance that thetr3.in covers in the first 60 seconds.

(b) Given that the train covers a total distance of 3000m dtuing the period
of retardation, calculate the value of I.

Answer (a) (i).....•.... m1~ [1]

(it) m [2]

(b) t :0 ••••••••••
~:'..... [3)

. [TUrD over I

175
Download more resources likeII this on ECOLEBOOKS.COM

28 The table shows the frequency distribution of the number of instruments that
each of the members of a band could play.

Number of instruments 1 2 3 4 5
..

Frequency x 10 5 y 1

(a) (i) Given that the total number of people in the band is 30, use this
information to form an equation in terms of x and y.

(li) Given further that the mean of the distribution is 2,1, form
another equation in terms of x and y.

- (b) Solve the two simultaneous equations.

Answer (a) (i) [2]

(ii) [2]

(b) X"" ••••••• ~ [3]


y •••....

4008111402&11 xzooo

176
Download more resources like this on ECOLEBOOKS.COM

MATHS NOVEMBER 2000

400811

POSSIBLE ANSWERS

15
1. (a) 1,47633 (b) (i) ~ Iii)
. 72 28

2. (a) 20 (b) 27

3. (a) 9 hours .(b)(i) 09 35 . (ii) 1835

4. (a) 6 (b) 0,6mor60em

5. . (a) (b) Bn Cf7'I A'

6. (a) 454 -(b) (i) 807 (ii) 47

b2
~7. (a). (b) -9or2
QC

8. (a) 2100 (b) . 26

9. (a) .20<' (b)(i) 1,3428x1Q24 (iQ 6.714x1017

10. (a) 910 or 9t.n (b) 4sor~ (e) 1025 or 102f,.."


. -

11. (a) (-14) (b)


(~3' 17)
8

12. (a) 30 (b) straight line passing through


(5;3) and (4;2) or (6;4)

13. (a) -1 (b) 3

177
Download more resources like this on ECOLEBOOKS.COM
.: .':,$ .:.. ~.-:.{'

14. ·.~i;'~22 ··.C?


(b) -142

(b) 224
15. :f~~{1:1~:;
.. <,:'
'. ".- (..)
II
3
-_ (b) 7:4 or '!_
4 4

X at 3 units along 08 (b) -2! -3Q . (c) Line DE drawn


.17.. ·4tl-.
/.:~,~~ from 0 and ending
followe9~ 2 units parallel
·toOA at 4 units to the
right of C
horizontally

(a) 67° (b) 23° (c) 87°


18.

19. (a} 195 (b) 3

20. (a) 1500 (b) -9

(a) . (b) (~ 234° (ii) 36~ (iii) 10


21: 2100

22. (a) (b) 13

I
(a) 2+18=20 (b) 3n (c) 2+3n=42
.23.
3n =40
40
n=- ..•
-'
:. n is not a whole
number or 40 is not
a multiple-of 3.

24. (a) 2376 (b) Ji16 or 6.[6

25. (a) 142000 (b) $213

178
Download more resources like this on ECOLEBOOKS.COM
26. (a) $57,80 (b) $15

27. (a)(i) % or 0,5 (ii) 1 500 (b) 360·

28. (a) (i) K+10+5+y+1=30 (ii) x+ 20+ J5+ 4y +5


30

(b) x=11,y=3 . x+20+J5+4y+5


or ----""--~ =2,1
x+y+16 .
EGI4OO8-1-nOO

179
Download more resources like this on ECOLEBOOKS.COM

180
Download more resources like this on ECOLEBOOKS.COM

ZIMBABWE SCHOOL EXAMINATIONS COUNCIL


General Certificate of Education Ordinary Level

MATHEMATICS 4008/2
PAPER 2

Thursday 9 NOVEMBER 2000 Morning ,2 hours 30 minutes

Additional materials:
Answer paper
Geometrical instruments 138850
Graph paper (3 sheets)
Mathematical tables
Plain paper (1 sheet)

'~ME 2 hours 30 minutes

INSTRUCTIONS TO CANDIDATES

Write your name, Centre number and candidate number in the spaces provided on the answer
paper/answer booklet.
Answer all questions in Section A and any three questions from Section B.
Write your answers on the separate answer paper provided.
If you use more thar lone sheet of paper fasten the sheets together. '
I

Electronic calculators must not be used.


All working must be clearly shown. It should be done on the same sheet as the rest of the
answer.
Omission of essential working will result in loss of marks. '
Iithe degree of accuracy is not specified in the question and if the answer is not exact, the
answer should be given to three significant figures. Answers in Jegrees should be given to
one decimal place.

INFORMATION FO_RCANDIDATES

The number of marks is given in brackets [ I at the end of each question or part question.
Mathematical tables may be used to evaluate explicit numerical expressions.

-~------------,.-,----------.-----------------
This question paper consists of 12 printed pages.

-:' ,PYf!ght: Z,'nL'l,Wt; S,:hO()! Examinations Council, 2000.


. ©ZIMSEC 2000 [Turn over

181 '
Download more resources like this on ECOLEBOOKS.COM
1

Section A [64 marks] .

Answer all the questions In this section.

1 (a) Simplify 4rr(3r - 1'). [2]

(b) Solve the equation 4(3%+ S) -7(6 - x) - 16. [2]

( c) Factorise completely

(I) r + 7i' + 12%,


. Ch") ab-ad-bc + cd.

(d) Express as a single fraction in its simplest form


y l' -3
r---+-. [3]
4y-.-1 S
f,
_".'

_(e) The time- taken to cook wkg of meat is given by the expIession (48w + 23) minutes.
Calculate how long it would take to cook 3kg of meat, giving yoW' answer in hours
and minutes. - [3]

182 _
Download more resources like this on ECOLEBOOKS.COM
3

2 (a)
A

In the diagram, BAD = BCn:;::90°, AB = AD. Be "" 7cm and cn "" tern.
Calculate the length of line AB. [4]

(b) A rhombus has sides oflength Scm and one of its ang'. ~is ,70°. Calculate

(i) the length of the longer diagonal of the rhombus,

(ii) t,:c area of the rhombus. [6]

3 <a> U = (4 -1)
0 2' V= 0
(2 3(').
(i) 'Find the inverse ofU.

(li) Given that UV = VU, calculate the value 'of t. [6]

(b) e = {x: 1 :;:;x < 10, x is an integer},

A= {x: x is a perfect square} and

B = {x: x is a muhiplc of3}.

(i) List the elements of A.

(ii) Find n(AnI~).

, (in) Write dov i.~ (/\ : !B)' by listing its clements: [51

400812 N2000

[Turn over

183
Download more resources like this on ECOLEBOOKS.COM
4
•• (a)

In the diagram. KL is a diameter of the circle KLMN. NM is parallel to KL and


MKL = 22°.·. Caiculate
\.

(i) KNM,
,. [4]
fu)MKN .

400IIl NlOOO

184
Download more resources like this on ECOLEBOOKS.COM
5
(b)

In the diagram, BP = BQ, PBA = CBQ anq BAQ = BCP.

(i) Name, in correct order, the triangle that is congru~t to triangle ABQ.
State the case of congruency.

(ii) Hence write down pairs of equal elements, not already given, in the
congruent triangles. [5]

5 (a) Find the average of the three numbers It, - 2t, 3f. [3]

. (b) A householder used 960 units of electricity for which the charges were 35,2 cents
per unit for the f rst !00 unit", 30,5 cents per unit for the next 200 units and 26 cents
per unit for the remainder. Calculate.the total cost of the electricity used by the
householder. [3]

(c) A greengrocer bought 30kg of bananas at $18,75 per kg. He sold 40% of them at
*
$20,50 per kg and 66 % of the remainder at $19,50 per kg. If the rest could not be
sold, find the percentage loss he made, giving your answer correct to 2 significant
figures. '

4(l0812 N2000

(Turn over

185
Download more resources like this on ECOLEBOOKS.COM
6

6 Answer the whole of this question on a sheet of plain paper.

Use ruler and compasses onlyfor all constructions and show clearly all the construction
lines and arcs.

(a) Construct on a single diagram

(i) triangle ABC in which BC = 10em, CA = 7 em and AB = 8,5 em,

(ii) 'the locus of a point P, such that BP = PC,

(iii) the locus of a point Q, such that AQ == 4cm. [6]

(b) Given that Q is on AC~measure and write down the length ofBQ. [1]

(c) Shade the region inside the triangle ABC, in which a point R lies given that

BR~RC
and AR~4 em. [2]

186
Download more resources like this on ECOLEBOOKS.COM
7
Section B [36 marks]

Answer thre« questions in this section.


'7 (a) Solve the equation 3,x2 - 5x - 4 = 0 giving the answers correct to 2 decimal places. [5J
. . 1 1 1
(b) GIven that -=-+-
gel'

express e in terms of f 8D:d g. . [4]

(e)' It is given thaty is inversely proportional to (x;- 3) and thaty = 4 when x = 2,


express y in terms of x. [3]

[Turn OH'!"

187
Download more resources like this8 on ECOLEBOOKS.COM

Q~l~m
._j.

-r i I

• !

T
11.
, ; . pi
.. '
..
. ,
! ; I

I '.

·~4
':.~ , I !

In the diagram, line I intersects the curve y = XI- - 6x + 8 at P and Q.

Use the diagram to answer the following questions.


Estimate, correct to 1 decimal place, the solution of x -6x + 2 = o. [2]
2
(a)

.. (b) 'Write dO\\TI the coordinates of P and the coordinates of Q. [2]

(c) rind the gradient of the line I and hence or otherwise write down the equation of I. [3]

(d) Write down the equation in x \vhi;:h has the x-ccordinatcs ofP and Q as its
solutions. gil, ingthc answer in .he form en: or- bx + c= 0, where a. band care
mtcgers. [3]

(c) Estimate the ..irca of the shaded region. [2]

188
Download more resources like this on ECOLEBOOKS.COM
9
9 ADSW'er the whole of this question on a single sheet of graph paper.

The masses of pupils in Form 1 at a certain school are given in the following table.

Mass (m)
in kg 35 <m~40 ·40<m~45 45 < m s 50 50_<m~55 55 <m s 60 60 <m~65
Number
of pupils 5 30 23 25 12 5 _j

(a) Copy and complete the following cumulative frequency table.

Mass (m) in kg
35 40 45 50 55 60 65
Number of pupils with this mass
or less 0 5 58 100

[2) _

(b) (i) Using a horizontal scale of 2cm to represent 5kg and a vertical scale of 2cm to
~
represent 10 pupils draw a smooth cumulative frequency curve for tins
distribution. (4)

(if) Showing your method clearly, use your graph to estimate the median of this
distribution. [2]

(e) One pupil isselected at random.

(i) Find as a fraction in its lowest terms the probability that the pupil's mass is more
than 45kg hut less than or equal to 55kg.

(ii) If the probability that the pupil's mass is more than m kg is J~' use the graph to
find the value ofm. [4]

4008/2 ~:OOf) [Turn over

189
Download more resources like this on ECOLEBOOKS.COM
10
10

In the diagram, E, F, G and H are four points on a sportsfield. E is due north of F, H is due east
ofF, F~H = 65°, FH = 15.2m, HO = 12,Sm and FGH = 75° 20'.

(a) Calculate

(i) the distance EF, [3]

..
GFH., [4] .
Cu")

CJiJ") the bear'ng of 0 from F. [2]

(b) Avemca1 pole, whose top is T, is erected at H. '

Given that the height of the pole is S,6m, calculate the angle of elevation ofT from F. [3)

400112 mooo

190 j
Download more resources like this on ECOLEBOOKS.COM
11
11 Answer the whole of this question on a single sheet of graph paper.

Triangle X has its vertices at (-4, 4), (-2, 2) and (0,4): Using a scale of Icm to represent 1 unit
on each axis, draw axes for the values of x and y in the ranges - 45 x ~ 14 arid - 6 ~ y ~ 12
respectively. .

(a) Draw and label triangle X... [1]

(b) A single transformation Z maps triangle X onto triangle Z (X) which has vertices at
(6, -6), (3, -3) and (0, -6).

(i) Draw and label triangle Z(X).


(li) Describe fully the transformation.Z. [4]

(c) The transformation T is a translation which maps triangle X onto triangle T(X) with
the vertex (-4,4) of triangle X mapped to the point (8,2).

(i) Write down the translation vector T.

(ii) Draw and label triangle T(X). [4]

(d) The transformation R is a clockwise rotation of 900 about (4, 2). Draw and label
triangle R(X). [3]

[Turn over

191
Download more resources like this on ECOLEBOOKS.COM
· .12

12 (a) List the integer values of x that satisfy the inequality

14 - x ~ 5x < 27. [3]

(b) Ratidzo has $200 pocket money. She decides to buy c crayons and b books.
Each crayon costs $12,50 and each book costs $25. She wishes to buy at least
5 crayons and at most 4 books.

Write down three inequalities, other than c ~ 0 and b ~ 0, which satisfy the above
conditions. [3]

(c)

In the diagram -OPA and OBR are straight lines. PR intersects AB at Q. 04. = tI,
.---.
OB =b,
.-......__. - -
OP = 4PA .and OR =30B.

(i) , Express in terms of a and/or b

(a) AD,

(b) PRo

-(ii) Given that PQ = hPR, -


express PQ in terms ofh, a and band hence, show that

-
OQ= t(1-h)a + 3hb. [6]

192
Download more resources like this on ECOLEBOOKS.COM

MATHS NOVEMBER 2000


4008-2

POSSIBLE ANSWERS

1. (a) 4rt2 (3r - r') = 12 r2t2 - 4rt5 B2

.(b) 12x + 20 - 42 + 7x = 16
12x + 7x = 1G + 42 - 20 MA1
x = 2 A1

(c) (i) x3 + 7x2 + 12x = x (x 2


+ 7x + 12) .
= x(x + 3) (x + 4) B2

(ii) a(b - d) -c(b - d) M1


(b - d) (a - c) A1

(d) 5~+3(4~-1l
5(4y-1) M1

17~- 3 A2
5(4y-1)

(e) Time = 48 x 3 + 23 M1
48 x 3 + 23
60 M1
•..• ..t •.•• "',

2 hrs 47 minutes A1
(14)

2. (a) BD =7+1 M1
2AB2 = BD2
AB2 = 50 M1
2

AB = .)50/2 M1
=5cm . 1\'

193
Download more resources like this on ECOLEBOOKS.COM
,,
(b) ,,
,,
,,
,,
,,
8

=
(i) x 8 Cos 35° Ml
Longer diagonal = 2 x 8 Cos 35° M1
= 13,11 em A1

Alternatively \ r
Longer diagonal = \j 82 + 82 -:- 2x8x8eos1100 M2

13.11em A1

(ii) Area of rhombus = 8 x 8 Sin 700 M2


= 60,14 em 2
A1

(10)

3. (a) (i) det U = 8- a= 8

u·1 = (~g "2 1) B2


·0 4

M1

x 12-/ (X 4
MAl
() 21 =] ()\
21

M1
12-t=4
A1
t = 8

194
Download more resources like this on ECOLEBOOKS.COM
(b ) E = 1., 2', 3', 4', 5:, 6·, 7', 8·, 9·, 10
(i) A = 1; 4; 9 B2
. B = 3; 6; 9

(jj) A n 8 = 9 81
n(A n 8) 1 =
:iii) (A u B)' = 2; 5; 7; 8 B2
(11 )

4a)

OR
KN M = 180° - (90° - 22°) M1

= 112° A1

M k N = 180° - (112° + 22°) M1

= 46° A1

(b) (i) f\ ABO = CBP B1


f\ SAA or AAS or 2 angles and a corresponding side
or Angle, angle side B1

(ii) AO = CP, AB = CB, AQB = CPB B3


.(9)

195
Download more resources like this on ECOLEBOOKS.COM

5.. (a) ·1 %._ 2 % + 3 ~ M1


8

5
7 _ 5 + 3- M1
8

1 14 - 20 + 29 M1
-
3
,
8

23 or 0,9583 A1
24

(b) Total Cost= 100 x 3,5,2 + 200 x 30,5 + (960 - 300) x 26 M2


. .:;:: $267,80

(c) Cost Price = 30 x 18,75 M1


z
40 x 30 x 20,5 + (30 - 40 x 30) x 66 3 x 19,5 M1
100 100 100

= $480

% Loss made = 30x 18,75-480 x 100 M1


30 x 18,75

= 1'5% A1
(11)

6. - Refer to diagram drawn.


(a) (i) Correct triangle with sides 10 ~ 0,2cm, 7 ~ 0,2 cm and
8,5 ~ 0,2 cm. ~1
Correct construction arcs T1

(ii) Ruled bisector of BC at least 3cm long B1


Correct construction arcs B1

(iii) Circle centre A with radius 4 .:!: 0,2 cm drawn C2

(b) BO = 8,7 cm B1
(c) Correct region bounded by solid lines inside the triangle,
shaded. R2
(9)

196
Download more resources like this on ECOLEBOOKS.COM

197
Download more resources like this on ECOLEBOOKS.COM

7, (a) 3x2 - 5 x - 4 = 0

x = ~b±\J b2 - 4ac
2a

=5 ~\j (~5}2- 4~3)(-4}


2x3

= 5'±G
6 81 81

= 2,26 Or - 0,59 81 81

Or by completing the square,

x2- §x=~
,3 3

(x_i)2
81

~ 6
81

x= '~'±~
6 36
81

= 2,26 or - 0,59 81 81

(b) 1= 1+ 1
9 e f

1 = 1.- 1 or ef = gf + eg M1
e 9 f

1= f - 9 or ef - eg = gf M1
e fg

e=~ ·A2
f-g

198
Download more resources like this on ECOLEBOOKS.COM
(c) ya _j_
x-3 M1

y=.JL_'
x-3

k= y(x-3)
= 4(2 - 3)
::::-4 A1
y=....±..
x-3 A1 .
(12)

8~ Refer to graph on question paper


(a) The values of x are read at the points where the line y =6
cuts the curve

x = 0;3 or 5;6 81 81

(b) P(1';3) Q(6;8) 81 81

(c) Grad of line I =1 . 81


The line passes through the point (1 ;3)
Equation of line is

y-3
X-1
= 1

Or y-3 =1(x-1) M1
y=x+2 A1

(Or using y = mx, + C, y = 1(x) + 2) M1


y=x+2 A2

(d) The x coordinates of P and Q are 1 and 6 respectively

Required equation is
(x ~ 1)(2 - 6) = 0 M1
x2 -7x+ 6 = 0 A2

(e) By counting squares


Area = 15,2 (units)2 82

(12)

199
Download more resources like this on ECOLEBOOKS.COM

Is 60
95 .
82

(b) Refer to graph


(i) , The 4 given points together with (45; 35), (55;83)
and (60;95) correctly plotted P3

Correct curve, smooth and not grossly thick. C1

(ii) Line y = h where 50 sh ~ 51 shown on graph. M1


=
Median 48,4·kg A1

(c) (i) p(45:S;; mass ~ 55) = 23 + 25 M1


100

= 12 A1
25

(ii) Line y = k where 90~ k1t 91 shown on graph. M1

m = 57,5kg A1
(12)

200
Download more resources like this on ECOLEBOOKS.COM

201
Download more resources like this on ECOLEBOOKS.COM

E
10. "

(a) (i) EF = tan 25° or 15,2 = tan 65°


15,2 EF
M2
EF = 15,2 tan 25° or 15.2
tan 65°

A1
= 7,088m

(ii) Sin G tH = Sin 75° 20'


12,5 15,2

Sin G tH = 12 5sin 75° 20'


M2
15,2

A2
G t- H = 52,7° OR 52° 43'
M2
(iii) "Bearing of G from F = 90° + 52,7
A1
= 142,7°

OR S 142,7° E

M2
(b) tan i- = _§&_
15.2
A1
( 11.)

202
Download more resources like this on ECOLEBOOKS.COM

++
--t--+.:-
, ,

.- -- ----' +-
.--~

,-+

_j_-
, _c

, I

,;

,- --

-- I I
I'

-Lt
...•..•.
-..f_ _
l'

!
r-
i 1. ~j
,..J-c...;,--+::
~
... ",
-'-
J.

- -,
._
"

--t-

_J +-_
,-i--
I
t-,

.- ..1

"
-
1-'
.-
.-

-- 'J. '
,4
, I
Ii
'...1.

_J:::_ ,.

.t
I
4-

I
'T
,-T;
I,

,--;-1

: -'--J---i-:--t
t+·~.

203
Download more resources like this on ECOLEBOOKS.COM

11. Refer to graph drawn.


(a) .6.X Correctly drawn t-1

(b) (i) L~Z(X) correctly drawn. M1

(H) Enlargement. M1
Center origin or (0,0) A1

Scale factor _11/2 or -1,5 A1

, (c) (i) Translation vector = (8 )_( -44)


2

B2
= (~:)
(ii) t- T(X) correctiv drawn with vertices at (8;2). (10;0)
and (12;2) T2

(ii) .6.R(X) correct; i drawn with vertices at (4;8), (6: 10)


and (6;6) .. R3
( 12)

12. (a) 14 - x .s 5x and 5x. < 27


14 .s 6x and x < 5 ~

2}3' < x and x 5 ~

values are 3; 4; 5 83

(b) 12,5c + 25b s 200


125c + 250b s 2 000
5c + 10b ~ 80 81
c ~ 5 81
b .s 4 81

204
Download more resources like this on ECOLEBOOKS.COM

206
Download more resources like this on ECOLEBOOKS.COM

(c) (i) (a) ---+ --4 ---+


AS = AO + OB
=a+b B1

.---+ ---+ ---+


(b) PR = PO + OR

4 82
- -a +3b
5
---+ 4
(ij) PQ = h(--a + 3b) 82
5 .
---+ ---+ ---+
OQ = OP+ PQ

44· 82
= --a + h(--a + 3b}
55·

4 4 .,
= -a --ha +3b B1
5 5

4 A1
= -(1-h)a+3b
5
(12)

!!P_

205
Download more resources like this on ECOLEBOOKS.COM
Candidate Name . Centre Number Candidate Number

II I
ZIMBABWE SCHOOL EXAMINATIONS COUNCil
General Certificate of Education Ordinary Level

MATHEMATICS 4008/1, 4028/1


PAPER 1

Monday 4 JUNE 2001 Morning 2 hours 30 minutes

Candidate, answer on the question paper.


Additional material:
Geometrical Instruments 1 ..,
.J..ti
3 -, ("
,. v
-' •

TIME 2 hours 30 minutes

INSTRUCTIONS TO CANDIDATES

Write your name, Centre number and candidate number in the spaces at the top
of this page.
Answer all questions.
Write your answers in the spaces provided on the question paper.
If working is needed for any question it must be shown in the space below that
questlon.
Omission of essential working will result in loss of marks.
Mathematical tables, slide rules and calculators may not be brought into the
examinatlor' room.

INFORMATION FOR CANDIDATES

The number of marks is given in brackets [ J at the end of each question or part
question.

FOR EXAMINER'S USE

This question paper consists of 17 printed pages and 3 blank pages.

Copyright: ZimbabwlI Scho.,>1Examinations C(Juncil. 200'1.

eZIMSEC 2001. [Turn over

207
Download more resources like this
2 on ECOLEBOOKS.COM
NEITHEfl MATHEMATiCAL TABLES NOR SLIDe RULES NOR
CALCULATORS MAY BE USED IN THIS PAPER ••
••'$
VII
1 (a) Giving your answer as a common fraction in its lowest terms,
find the value of

(i) 71
5
-61
3 '
5
(ii) 0,45 of -.
9

(b) Express -15 as a terminatmq


.. d ecimar,
. I
16

-Answer (a) tn [1]

(ii) [1 ]

(b) ----- [1]


~\
...
~ (a) Express..158,697

. (i) correct to the nearest hundredth,


(ii) ",orrect to two significant figures.

(b) ExpreSfS720 as a product of its prime factors.

Answer (a) (i) [1]


- ~',~~{'.

(ii) (1)

(b) [1]

4008/1 4028/1 J2001

208
Download more resources like this on ECOLEBOOKS.COM
3
3 .(a) Find the sum of 2 days 15 hours and 3 days 21 hours,
giving your answer in days and hours.

(b) Express 345 as a number in base two.

Answer (a) ------------ days ---------hours [1]

(1)) ------------------------------------- [~]

\
2 b:2 b :2
" I"tv
SImp I
a - :2 + a - 3a
ab+a .2a.

Answer ------------------------ [31

4008/1 4028/1 J2001


[Turn over

209
Download more resources like this on ECOLEBOOKS.COM
4

8 (a) Two similar cylinders have their volumes in the ratio 1:8. FIr
Write down the ratio of the areas of their circular surfaces. I' '..,
VII

North

A, Band C are three points on level ground.


The bearing of C from B is 138° and ABC = 92°.

Calculate the bearing of B from A.

Answer (s) -------- [1]

(bJ------- [2]

400811 402811 J2001

210
Download more resources like this on ECOLEBOOKS.COM
6
8 Evaluate
p,,.;,...
(a)

I .

(b) - 27;.

Answer (a) -------- [1]

(b) [21

·9

In the diagram,(ABE and DCF are parallel straight lines, ABCD is a


parallelogram, bc= 7cm, EC = Scm and ECF = 65°.
Using the infotmation given below where necessary,find,

(a) the are~ of 6DEC,


(b) the are. ~f parallelogram ABCD.

[sin65° = 0,91; cos65° = 0,42; tan65° = 2,14.)

Answer (a) --- : .m~. .. cm2 [2]

(b) -- ---_--cm2 [1]

4008/1 4028/1 J2001

212
Download more resources like
7"
this on ECOLEBOOKS.COM

10
Bu.
u.
'-r-

In ~he diagram, line I, and line 12 are parallel.

(a) Find the gradient of line ',.


(b) Find the equation of line '2'
giving your answer in the form
ax + by = c where a, band c are integers.

Answer (a) --------. ---' ---:- [1]

(b) ------ [2J

4008/1 4028/1 J2001


[Turn over

"

213
Download more resources like this
8 on ECOLEBOOKS.COM

. i*
Write down the three Inequalities which define the unshaded region in
.the diagram above.

Answer ---_._-- (1]

(1]

--.---- [1]

4008/1 402811 J2001

214
Download more resources like 9this on ECOLEBOOKS.COM Par
12
c UIO

In the ciiagram, AB = 11 em, BC = 7em and AC = 6cm.


Calculate the value of cos ACB , giving your answer as a common
fraction in its lowest terms.

Answer cos ACB = ------------ [31

13 Solve the simultaneous equations

2x -3y = 1::$,
3x + 2y = O.

Answer x = -------------------
y = -------------------- [3]

4008/1 4028/1 J2001


[Turn over

215
Download more resources like this
10 on ECOLEBOOKS.COM

14 (a) Estimate, to the nearest whole number, the value of

7,9x .J8O,6
1,8 x 3,1

(b) The lengt" of a square is given as 1Oemcorrect to one


significant figure.. Find the smallest possible area of the
square.

Answer faJ -------------------------_:- [2]

(b) -------------------..,,,----cm2 [2]

-5
15 Given that m = 4,8 x 103 and n = 1,2 x 10 ,

find, giving your answer in standard form,

(a)
(b)

(c)

Answer (a) ---------------------- [1]

(b) -------------------- [1]

(c) ----------------------- 121

4008/1 4028/1 J2001

216
Download more resources like
,11 this on ECOLEBOOKS.COM
16 Ca) Expandand simplify (x + 2){5 + 2x - x2).

(b) Factorise completely

3x2 - 6x - ax + 2a.

Answer (a) ---------------------~---------- [2]

!b) .---------------.---------------. f 21 .

17

. ,;B
In the diagram, P.A.and p~~~
are tanqents Lu the circle centre O.
Given that Q is a point on the mil lor arc AS and that AOB = 1400,
find

(a) OBP,
~
(b) APB,
Ce) AQB.

.::i/Iswer ,'J) -----.------.- - ,--- .... -- [1 J

(.0) ------ -"--.- ...-." ._..---- .. - .. __ lj !

I•••');J

:.Turn over

217
Download more resources like this on ECOLEBOOKS.COM
12
Eumlna'l
18' A= (: ~~). B = (~~ ~) and M = (-~-:) Find

(a) AS,
(b) the inverse of M.

Answer Iel ------------------ [2]

- rb) -------------- [2]

19 It is given that a = (~~} Jfi = 4,123 and,[0 = 1,304.

Find
(a) ~O,17,
(b)' the magnitude of a correct to two significant figures.

Answer (a) ------------:-----------:.....--- [2]

(b) -----------~---------------~ [2]

4008/1 4028/~ J200i

218
Download more resources like
13 this on ECOLEBOOKS.COM
For
20 In a regular polygon of n sides, the size of each interior angle xo, is Runrina'a
given by the formula Use

x = 180 - 360.
n

(a) Make n the subject of the formula.


(b) Calculate the number of sides of a regular polygon whose
interior angle is 1so-,

Answer (a) n = --_. ----------- [2]

(1») ------------~-------------------- [2]

4008/1 4028i1 J2001


(Turn over

219
Download more resources like this on ECOLEBOOKS.COM
14
y i I
21 ,I 1 1 Ii F«
BJIDIina'I
4 '
i
I ! U.
. __ -- f-. I

2
I
I
! I -
-"
~4 A

I ~~.
~2 ~ 0 D!

-2 -tifJ+-- ~E i ~}
;
_--t-~~~-+----I
x

iL~+-_-+-~ "'-----I~_
c
_ ,-,-:--tl I
.
I
I
I
'
i
'-
F
I
-4+----i-i--i---~ ;
1 1 I I II
The diaqram shows AABC and bDEF.

(a) On the diagram, draw and label ~PQR, the imllge of ilABC
. ~~,
under a translation
.
of vector 'i '
f\.4)
(b) Describe .fully the :single transformation that maps AABC onto ilDEF.

Answer is) On the diagram. . [1]

(b) ------. _...-----...--------- -----

-.._------,,--.--
--~-------------- [3]

4008/1 402811 J2001

220
Download more resources like this on ECOLEBOOKS.COM
15

22 The scale of a map is given as 1: 50 000. For


ExlMDinc:ta
Usc
fa} Calculate the length of a line on the map that represents a road
6,4km long.

(b) Calculate the area, in hectares, of a piece of land represented on the


map by an area of 3cm2 •

[1 hectare = 10 OOOm2.1

Answer (a) ----------------------------'----cm [2]

(b) --------------------------------ha [3]

23 m Is directly proportional to n2 and m = 28 when n = 2.

(a) \I\/rite down an expression for m in terms of n and a constant k .

. (b) Calculate

(i)the value of k,
(ii) the values of n when.rn = 63.

-..._,
\

Answer fa) -------------------------- [1 )

(b) mk = ----~--------.,----[21

(ii) n = ----- or ------ [2)

4008/1 402811 J2001


[Turn over

221
16 this on ECOLEBOOKS.COM
Download more resources like
\

. 24 Given that log216 = 2,334 and log3 = 0,477, find


RUn'••.•••

(a) log2,16,
(b) log6,
(e) log72.

Answer (a) ------------ [11

(b) --- [21

(c) ------- [2]

25 (a) The graph of x + 2y + p = 0 and the graph of


3 x + qy = 9 intersect at the point (-2, 3). Find the value
of p and the value of q.

(b) Factorise4x2-12x + 9.

Hence or otherwise solve the equation 4 x 2 - 12 x + 9 = 16.

Answer (a) p = ----------------- [1)

q :;:------------------ [1]

(b) ------------------- [2]

x = ---------- or ----------- [2]

4008/1 4028/1 J2001

222
- - ._-------
Download more resources like this on ECOLEBOOKS.COM
---------------------

17
26
Rz''''jcn'a
--------------~-- .' . UIO

Velocity
(kmIh)

30

5 15 20 Tnne(min)

The diagram shows the velocity-time araph of a car during a period


of 20 minutes. Find

(a) the retardation during the first 5 minutes,


(b) the dist-ance, in kilometres, travelled a~ constant elocity,
(e) the total distance travelled, in kilometres,
(d) the average velocity, in krn/h, during the 20 minutes.

~nswer fa) --------- --- kmlh/min [1]

(b) ------------------km [2]

(e) ------------------------------- ----km [21

(d) ---------------------. ----km/h [31

400811 4028/1 J2001


Download more resources like this on ECOLEBOOKS.COM
18
BLANK PAGE

4008/1 4028/1 J2001

224
Download more resources like this on ECOLEBOOKS.COM

MATHS JUNE 2001

4008/1

POSSIBLE ANSWERS

6. (a) 1, 3 3.Jl3
(b) -or--
m 13

7. 55 x+y
(a) - (b)
89 x+2y

8. (a) 25 (b) -9

9. (a) 25,48or25-48 (b) 50,96or50-96 or50-24
100 100 25
12
or2525

10. (a) -4 1
-or-I- (b) 4x+3y=32 or
3 3
-4x-3y=-32

225
-------------------
Download more resources like this on ECOLEBOOKS.COM

11. y>-4 or -4<y


2y :s7x+14 or 7x+14~ 2y
- y+x '5:.7or 7~ y+x

-3
12.
7

13. x=2 y=-3

(b) 90,25 or 90 25 or 9Oy..


14. (a) 12 100

9
(b) 1,44x10·1O (c) 25x10·
,
15, (a) 2,4x104

16. (a) 10+9 x - _i3 or (b) (x-2)(3x-a)


equivalent expression or(3x-a )(x-2)

(b) 40° (c) 110°


17. (a) 90°
5 \
18. \(a)
(~~J (b)
-i (~2 -3)

19. (a) 0,4123 (b) 13

360 or -360 (b) .12


20. (a)
180-x x-I80

, 21. (a) ~ PQRwith (b) Shear, factor -3


P (4;3) x -axis or line y=o
Q (6;30) and being invariant
R (6;1)

8 (b) 75
22. " (a) 12,8 or 12-
10
4
or 12-
5

n2 I 30r-3
m=kn2or - (b) . k=7or- (c)
23. (a) 7
k
with respect'
to (a)

226
Download more resources like this on ECOLEBOOKS.COM
24. (a) 0,334 (b) 0,778 (c) 1,857

25. (a) p=4 q=5 (b) (2x-3)(2x-3) or (2x-3)2


x=-% or 3% or.
x=-O,5 or 3,5
(2 is accepted in place of 3% or3,5)
2

26. (a) 12 (b) 5 (c) 15 (d) 45

EGfC:4008-1-J01

227
Download more resources like this on ECOLEBOOKS.COM

228
Download more resources like this on ECOLEBOOKS.COM

ZIMBABWE SCHOOL EXAMINATIONS COUNCIL


General Certificate of Education Ordinary Level

MATHEMATICS 4028/2
PAPER 2

Thursday 7 JUNE 2001 Morning 2 hours 30 minutes


Additional materials:
Ans\yerpaper
. Geometrical instruments
Graph paper (3 sheets)
Mathematical tables 02879 -
Plain paper (I sheet)

TIME 2 hours 30 minutes

INSTRUCTIONS TO CANDIDATES

.Write your name, Centre number and candidate number in the spaces provided on the answer paper/answer
booklet.
Answer all questions in Section A and any three questions from Section B.
Write your answers on the separate answer paper provided.
If you use more than one sheet of paper, fasten the sheets together.
All working must be clearly shown. It should be done on the same sheet as the rest of the answer.
Omission of essential working will result in loss of marks.
If the degree of accuracy is not specified in the question and if the answer is not «act, the answer should
be given to three significant figures. Answers in degrees should be given to one decimal place.

INFORMATION F0R CANDIDATES

The number of mark; is given in brackets [ ] at the end of each que~on or part question.
Mathematical tablesror electronic calculators may be used to evaluate explicit numerical expressions.

This question paper consists of 11 printed pages and 1 blank page.

Copyright: Zimbabwe School Examinations Council, 2001.

OZlMSEC 2001 (Turk over

229
Download more resources like this on ECOLEBOOKS.COM
2
Section A [64 marks]

Answer all the questions in this section.

1 (a) If 18 200m2 of land is shared in the ratio 2:3:8, calculate the area of the smallest
share. [2]

(b) Given that x ;;::29 t. state the least possible value ofx if

(i) x is a perfect cube,


(ii) x is a rational number. [2]

(c) Calculate the time that $2000 would take to amount to $3645 ifinvested
at 23 t % per annum simple interest. [3]

(d). A girl bought 72g of chocolates for $10,80. Calculate the amount she would
have paid for O,5kg of the same type of chocolates. [2]

2 (a) Given that a = 3, b =- 2 and c = 10, calculate

(i) ac',
(ii) abo [3]

(b) Express -4 - -- 3 as a smg


. Ie fracti .. Its SImp
ction m . 1est &.Lorm. [3]
p 1-5p

(e) Given that f (x) = 2x - 3, calculate the value ofk when f (k) = - 21. [2]

(d)

Q
In the diagram, MNP is parallel to CD. Angle PND= 59°, angleNMC = uO,
angle MCD = VO and angle CDQ = ~. ..
(i) Calculate the value of f.
(ii) Express v in terms of u. [3]

402812 J200 1

230
Download more resources like this on ECOLEBOOKS.COM
3

3. (a) A man earns $30 per hour for the first 35 hours and $40 foreach additional hour
that he works each week. Ifhe earned $1290 in a week, calculate the total
number of hours he worked.: [4]

(b)

27

B
The figure shows a uniform triangular prism, 27cm long. The righsangled
triangle PQR is a cross-section of the prism and PQ = RQ.=:=6cm and
angle PQR = 90°.
Calculate

(i) PRo
(ii) the volume of the prism,
(iii) the total surface area of the prism. [7]

4 Answer the whole of this question on a sheet of plain paper.

Use ruler and compasses onlyfor all constructions and show clearly all the
construction lines and arcs. .

(a) Construct on a single diagram

(i) a parallelogram ABCD with AB = 9cm, AD :: 7cm and the angle


BAD = 60°,
(ii) . the locus of points equidistant from AB and AD,
(iii) the perpendicular bisector of Be. [7]

(b) (i) Mark the point P that lies on DC and is equidistant from AB and AD.
(ii) - -Measure and write down the length ofBP. [2]

(e) Describe the locus that the perpendicular bisector ofBC represents. [2]

402812 J2001 [Turn over

231
Download more resources like this on ECOLEBOOKS.COM
4
5 Eachof37 pupils when ordering an ice cream-had at least one of the following flavours:

a vanilla,
a chocolate,
a peppermint.

In the Venn diagram. the expressions represent the number of pupils in each subset.

Vanilla . Chocolate Peppermint

(a) Write down the total number of pupils who chose a peppermint. [1]

(b) Express y in terms of x in its simplest form. [2]

(e) The number of pupils who chose a peppermint only was three more than the
number who chose chocolate only.

(i) Write down an equation in x.


(li) Solve this equation.
(iii) Determine the number of pupils who had a chocolate only. [4}

(d) Find the probability that a pupil who had a anilla also had a chocolate. [2]

(e) Two pupils were chosen at random from all the pupils,
Calculate the probability that both pupils had peppermint,
giving your answer as a fraction in its lowest terms. [3]

402812 1200 I

232
~ Download more resources like this on ECOLEBOOKS.COM
5
6 Answer the whole of this question on a single s beet of graph paper.

The charges made by a firm of carriers named Ph'iri Packages, for delivering parcels of
various masses are based on a fixed minimum char ge for parcels of 5 kg or less. For
parcels over 5 kg the total charge is made up of the fixed minimum charge together with
an additional charge which is proportional to the amount by which the mass exceeds 5 kg.
For parcels over 5 kg the table shows some exampl es of the charges.

Mass of parcel (kg) 15 23 29

Charge ($) 28,50 40,50 49,50

(a) Taking 2 em to represent 5 kg on the horizo ntal axis and 2 em to represent


$5 on the vertical axis, draw a straight line graph to show the total
charges for parcels up to 30 kg. [4]

(b) Use your graph to estimate

(i) the fixed minimum charge,


(ii) the rate per kilogram for the amount by which a parcel exceeds 5 kg,
(iii) the m~s of the parcel that would co st $38;50 to dispatch. [4]

(e) Calculate the charge for a parcel of mass 75 kg. [2]

'01812 J2001 (Turn over


.

233
Download more resources like this on ECOLEBOOKS.COM
6
Section B [36 marks]

Answer thru questions in this section.

IB.It••qHltlo. take 1C to be 3,1"42.

In the diagram, ABeD is a rectangle and PQRS is a sector of a circle centre P.


Given 'that
. P is also the mid-point of AB, PQ = PS =. 8 em and the length of the
arc QRS- 14cm,calculate

<a) th~ perimeter of the sector PQRS, [1)

(b) the angle SPQ. [3}-

(e) -tho area of the sector PQRS, # [2]

(eI) the area of the rectangle, {4J

(e> tho area of the shaded region. [lJ

40lIIl rzoo I

234
Download more resources like7 this on ECOLEBOOKS.COM
8

ID the diqram, ABC is a triangle in which BC - 9cm, angle ABC - 68' - ••••••
BAC ~ 42'.

(a) Calculate the length of AC. [3]

(b)' The point H is on AC such that AH - 4,lcm and the point G is OIl ABsuch that
AG ~ 4.7cm. Calculate

(I). . the length ofHG, (4]


(Ii) the area of the triangle AHO. (3]

(e) Calculate th~ perpendicular distance ftom B to AC. [l)

(Tura 8ftI'

235
Download more resources like this8 on ECOLEBOOKS.COM
'9

The diagram shows a circle with diameter CB. ED and CB produced meet at A. TC is the tangent
to the circle at C, and the angle TCE = 35°.

(a) Calculate

(i) angle CBE,


(ii) angle BeE,
(iii) angle BDA. [3]

(b) ,Name, in the correct order, the triangle that is similar to triangle ADB. [1]
'_
(c) IUs given that AB =(x + 3) em, CE =x em, AE = 8 em and BD = tern.

(i) Show that x' + 3x - 8 = o.


(ll) Solve this equation, giving the answers correct to 2 significant figures.
, (iii) Hence write;down the length ofEe. [8]
'#

4028/2 J2001

236
Download more resources like this on ECOLEBOOKS.COM
10
11 - Answer the wholt: ofthisquestion on a single sheet of graph paper.

Using a scale of 2cm to represent 1unit on each axis, draw axes for values
of x and y in the ranges-3 ~ x 55 and -6 5 ys: 4 respectively.

(a) Triangle ABC has vertices at (1,1), (3,1) and (2,3) respectively. Draw and label
triangle ABC. [1]

. (b) Triangle AIBIC1 is the image of triangle ABC under a reflection in tbey- axis.
Draw and label triangle AlBICI. [2]

(e) Triangle A2B2C2 is the image of triangle ABC under an enlargement scale
factor -2 and centre (2, 0). Draw and label triangle .-\B2~' [3]

(d) Triangle A3B3C3 has vertices at (-I, -3), (-1, -1) and (-3,-2) respectively.

(ii) Triangle A3B3C3 is the image of triangle ABC under a certain single
transformation. Describe fully this transformation. [4]

(e) Find the matrix that transforms triangle ABC onto triangle A,.B"C. which has the
.coordinates (1, -2), (3, -2) and (2, -6) respectively. [2]

402812 J2oo1

238
Download more resources like11 this on ECOLEBOOKS.COM
12 (a)

In the diagram, the point P is such that AP= 2PB and the point Q is' such thatAB = BQ.
Given that OA = a and OB = b, express as simply as possible, in terms of a and b

(i) AB,
(il) AP ,
(iii) OP,
(iv) AQ. [5)

• f J
(b) M is a 2 X 2 matrix such that M- 2f FindM. [3]
~O

(e)
"
N ;~ a 2 x 2 matrix such that N
(1~ =, (_.,'1-)' and Nl"(0~I = l~)'
( x \,
o \. 1 1)" ,/...

(i) Find the matrix N.

(ii)" Find the value ofx ifN has no inverse. [4]

4028/2 J20() I

239 "
Download more resources like this on ECOLEBOOKS.COM
12
BLANK PAGE

4028121200

240
Download more resources like this on ECOLEBOOKS.COM
MATHEMATICS JUNE 2001

400814028102

POSSIBLE ANSWERS

1. (a) Smallestshare = ~ x 18200 M1


13

241
Download more resources like this on ECOLEBOOKS.COM

r = 180°_59°
,._ = 121° 81

(ii) • u +v = 180 0
(allied angles on /I Jines)" M1

v = 180-u A1

[111

3 (a) Amount for first 35 hrs 30 x 35 = M1


Remaining amount = 1 290 - 30 x 35
No. of hours at $40 =
1 290 -30 x 35 M1
40

= 6 A1

Total No. of hours = 35 +6


"
= 41 A1

Ml

= 8,485 cm A1
(ii) Volume of prism = base area x height

::(t x6x6) x 27 M1

A1

(Hf) Total surface Area '


=2 x (%x6x6) +2 x (27x6)+ 27 x 8,485 M2
589,1 cor A2
[11]

243
Download more resources like this on ECOLEBOOKS.COM

4. Diagram

, I

~
(a) (i) Angle A = 60 ±2°
o
with correct construction arcs. C1

, Parallelogram with sides. 9,0 ±0,2 cm, and


7,0 ±0,2 cm and angles 60o±2° and 120o±2°. P1
Construction arcs for /I gram . A1

(ii) Ruled bisector of angle at least 3 cm long. L1

Correct construction arcs C1

(iii) Ruled perpendicular bisector of BC at least


3 cm long. L1

Correct construction arcs C1

(b) (i) P mar1<edon the intersection of DC and t~


angle bisector P1

(ii) BP = 6,2 ±0,2 cm ' B1

(c) Points that are equidistant from 8 and C 82


[11]

244
Download more resources like this on ECOLEBOOKS.COM

5. (a) n(P) = x +. 21-x


=21
BI
(b)" y+2+2x+x+21-x == 37
MI
y+2x=14
y ;;:::14-x
A1
(c) (I) 21 -x -3 = 2x 81
(ii) 3x= 18 MI
x==6 A1
(iv) n(chocolate only) = 2x
= 12 81
(d) p(VnC) = 2 82
37

(e) p{both had peppermint)= 21 x 20


37 36 M2
f'

== 35 A1
III

6. [12J
Refer to graph.
(a) The 3 given points correctly plotted P3
Straight line not grossly thick passing through
The correct 3 points L1
(b) (0 ,$13,50 81
(ii) , 40,S-28,S
23-15 or figures coming from the
graph M1

$1,50 A1
(iii) 21,S kg 81
(c) 13,5+ (.75,-5) x 1,5 M1
$118,50 A1
Note
With no graph the score is zero.
(10)

245
Download more resources like this on ECOLEBOOKS.COM

L
-'--+-

'- -·N· =t-+++t-t


I-H-+++ .L

246
Download more resources like this on ECOLEBOOKS.COM
(a) Perimeter = 8+8+ 14
= 30 em 81

SPQ
(b) _ ... xZn x 8 ··14 M1
3600

~ -I 14 x 3600
SPQ = - - ... .. M1
:2rc x 8

=100,2 C
or 100 0
15'

(c) Area of sector = .100,2 x 8 x 8 1t


360

= 56,01 ern" A1

(d)

AD =r= 8 cm

x= 8 Sin 50,1 or 8 Cos 39,9


0 0
M1

247
Download more resources like this on ECOLEBOOKS.COM
ThereforeUS - 2 x 8 Sin 50,1 "or 2 x 8 Cos 39,9° M1
Area = 8 x 2 x 8 Sin 50,1° M1
= 98,23 em2 A1

(e) Shaded area = 98,23 -56,01 M1


= 42,22 em2 A1

[12]

AC 9
8 (a)
Sin68 °
= Sin42°

9sin68° M2
AC=
sin42°

= 12,47 em A1

(b) (i) HG2 = 4,12+4,72- 2 x (4, 1)x(4,7)xCos 42° M1

= 10,26 A1

HG =~10,26 M1

= 3, 203 em A~

(ii) Area of MHG =%(4,1 )(4,7) Sin 42° M2

= 6,447 em2 A1

lL = Sin (180°- 68°_ 42°) M1


9

248
Download more resources like this on ECOLEBOOKS.COM

M1

.:;:8.451 ern A1

[12J

9 (a) (i) CBE = 35(anqieinalternatesegment) 81

(ii) BeE = 90"-35"


= 55° 81
A ,

(iii) 80/\ == BeE

(ext. L of cyclic quad equal to int. opp. L)

= 55° 81

(b)

-:
/
---- -------
..

Equal angles are marked the same.


LlACE is similar to L.\ADB . 81'

(c) (i) From the similar /1s,

t\B :: BQ
AE EC

249
Download more resources like this on ECOLEBOOKS.COM

x+3 = 1 M1
8 x

x2 + 3x -8 ==0 A1
-b±~b2-4ac
(ii) x
2a

-3±~32 -4(1)(-8)
2xl

= -3 ± J41 BIBI
2 Bl

x =1,7 81

or - 4,7 81

Alternatively, using. completion of the square,

( 3)2 =8+"2. (3)2


. X+"2

8181

81

= 1,7 81

or -4,7 81

(iii) EC =x
=1,7 cm 81
[12]

250
Download more resources like this on ECOLEBOOKS.COM

10. (a) rn =6 81

n = 1,5 81

Refer to graph

(b) The 5 given points and (1;6) and (4;1 ,5) correctly
plotted P3
Smooth curve, not grossly thick passing through the correct
7 points. C1

(c) 1,7 < x< 5,7 82

(d) Table of values for x +y = 5 or intercepts with graph


Marked or calculated. 'M1

Line x + y =5 correctly drawn A1

(e) Line parallel to x + Y =5 touching the curve at one point M1


x =2,2 A1

[12J

251
Download more resources like this on ECOLEBOOKS.COM

252
Download more resources like this on ECOLEBOOKS.COM

(a) AABC correctlydrawn .11

(b) AA1B1C1correctlydrawnwith verticesat (-1;1),(-3;1)


and(-2;3) 82

(c) AA2B2C2correctlydrawn
with verticesat (-1;1),-3;1)
and(-2;3) E3

(ii) Rotation M1

90· anticlockwiseor 270· clockwise A1


Centreof rotation (2; -2) A1

(f) Let matnxbe. (ac . bJ d

0+ b =1
30 +b =3 :.20 = 2 and 0=1
:.b =0

e+d= -2
3c +d+ -2:.2c = 0 and c = 0
:.d=-2

matrixis (10-2OJ 82

[121

253
Download more resources like this on ECOLEBOOKS.COM

254
Download more resources like this on ECOLEBOOKS.COM
12. (a), (i~ -~-
. AB=AO+oB

= -a + b
81
(ii) AP==fAB

= t(- a+b)
81

(iii) -~-
OP==OA+AP

=a+t(-a+b)
M1

=ta+tb
A1
(iV) -
AQ=2(AB) -
= 2(-a+b)
81
(b) M=3C -I ~) -I)
+ 2(~ 1.

= e '1)
-3 2 83

(c) (i)
Let N = (~ ~)

= (: :) G) = (-~)
= (:) = (-~J

Therefore

255
Download more resources like this on ECOLEBOOKS.COM

82
..
(ii) det N = -3(2)+ 1(x) =0 . M1

x=6 A1

[121
.~21lJ01

256
Download more resources like this on ECOLEBOOKS.COM

Candidate Name Centre number Candidate Number

ZIMBABWE SCHOOL EXAMINATIONS COUNCIL


General Certificate of Education Ordinary Level

MATHEMATICS 4008/1,4028/1
PAPER 1

Monday 3 DECEMBER 2001 Morning 2 hours 30 minutes

Candidates answer on question paper.


Additional materials
Geometrical Instruments
167338
TIME 2 hours 30 minutes

INSTRUCTIONS TO CANDIDATES
;

Write your name, Centre number and candidate number in the spaces at the top of this page

Answer all questions.

Write your answer in the spaces provided on the question paper

If working is needed for any question it must be shown in the space below that question ..

Omission of essential working will result in loss of marks

Mathematical tables, slide rules and calculators should not be brought to the examination
room.

INFORMATION FOR CANDIDATES

The number of marks is given in brackets[ ] at the end of each question or part question.

FOR EXAMINER'S USE

This question paper consists of 24 printed pages.

Copyright: Zimbabwe School Examination Council, N2001.


© ZIMSEC N2001 [Turn over

257
Download more resources like this on ECOLEBOOKS.COM

NEITHER MATHEMATICAL TABLES NOR SLIDE RULES NOR For


CALCULATORS MAY BE USED IN THIS PAPER Exarnile(s
Use
1 Evaluate, giving your answers as common fractions in their lowest terms

3 1
(a) -+-
4 5'

5 22
(b) -x-
8 . 45'

1 11
(c)
3 24

Answer (a) [1]

(b) [1]

(c) .: [1]

2 (a) Find 15% of $270.

(b) Express 93m as a decimal fraction of 3 km.

Answer (a) [1]

(b) [2]

4008/1402811 N2001

258 .
Download more resources like this on ECOLEBOOKS.COM
3

Calculate, in decimal form, the exact value of For


ExamIler's
Use
(a) 0,02 x 0,6,

(b) ~0,000004,

(c) 5,6-7,3.

Answer (a) ['1]

(b) [1]

(c) , [1]

4 (a) Evaluate (i)

(b) " 5x
Given that = 125, find the value ofx.

Answer (a) (i) '.' [1]

(ii) [1]

(b)x= [l]

400811 4028/1 N2001


[Turn over

259
Download more resources like
4
this on ECOLEBOOKS.COM

5 (a) . Ifx == m + n and j=r.m ~ n, express 3x + 2y I.Examinets


For
in tenus of m and n. Use

(b) Factorise completely 5r2- 5ri +qr - qt

I
1
!

Answer (0) ' ..:..: [1]

________ ---------.~~ [2] I


I
II
I
I
I.

II
I
I
40011/1 -40281 i N20C-! I

260
Download more resources like 5 this on ECOLEBOOKS.COM

6 " For
&IIi •••
U.

o
A, B, C, D and E'are points on a circle. Given that AB = BC = CD, DAB = 24°
and AEB = 17°, find
~
(a) ECD,
A

(b) BED,
A

(c) BAE.

Answer (a) ECD = ° [1]

(b) BED = °[1]

(c) BAE =: 0[1]

4008114028/1 N2001

261
Download more resources like 6 this on ECOLEBOOKS.COM

7 (a)' Express 0,0148 correct to one significant figure.


I~Use
(b) The sides of a rectangle are 15 em and 7 em measured correct to the
nearest centimetre. Find the smallest possible area of the rectangle.

Answer (a) :. [I]

(b) cm2[2]

8 <a) Solve the inequality 4 + 3x < 25.

(b) Expand Xl y (X4 =xy").

(e) Express 37 100 in standard form.

Answer (a} ...........•............ [1]

(b) ~ [1]

(c) ..................•..... [1]

400&11 4C2&11 NlOOI

262
--- -----------------~--------------------------------
Download more resources like this on ECOLEBOOKS.COM
7

9 <a) Evaluate 341s - 235, giving your answer in base 5.

(b) Jabulani completed a track event in 16,45 minutes.


/

(l) EXpress this time in minutes and seconds.

(ii) IfFarai took 1minute 56 seconds longer than Jabulani,


calculate the time he took to complete the event

Answer (a) , [1]

(b)(i) minutes seconds [1]

(ii) minutes seconds [1]

10 A bus has 80 passengers consisting of adults and school children.

<a) Given that the ratio of the adults to school children is 2 : 3,


calculate the number of adults on the;bus.

(b) Adult passengers pay $15 while school children pay half
the fare. Calculate the total amount paid by the passengers.

Answer (a) [1]

(b)$ ·····L2]

4()08'1 40::!S/l N2C:) 1


[Turn over

263
Download more resources like
8 this on ECOLEBOOKS.COM

11 There are 25 children in a class. Of these, 9 are in the


debating club and 13 are in the history club.

It is giventhat ~ = {children in the class},


D = {children in the debating club}, .
H = {children in the history club}

and n(D,", H) = 5.

(a) Find (i) the number of children in the debating


club only,

(ii) n(Du H).

(b) Express in set notation children who are in neither the debating club
nor the history club.

Answer (a)(i) [1]

(ii)n(Dv H)= ~ [1)

(b) ................................•.. [1]

400.,1 402111 N2001

264
----_. ---- ---------------------
Download more resources like this on ECOLEBOOKS.COM
9

-.
12 It is given that C = a + KN2 .
Find the two possible values ofN given that C = 102, a = 27 and K = 3.

Answer N = : or ~ [3]

_13 An incomplete water bill for a household is shown below.

Previous Present Consumption Rate $


Description reading reading (Units) cents/unit

14 May 27 June

Water 5992,4 6034,6 350 y

Fixed Charge 50,00

Amount z

Find the value of (a) x,

(b) y,

(c) z.

..
Answer (a) x = , ,.., [1]

(b) y = [1]

(c) z = [1]

400111 402111 N2001


(Turnover

-0"

265
10
Download more resources like this on ECOLEBOOKS.COM
14

.•••
1'

10 .

9 ...---0-
8

number 6 -
of
adults '5

4
~
3

2 .---
1

o 5 6
'0

7
shoe size
8

9
~

In a $UIVey,the adults in a literacy class were each asked their shoe size,
The bar chart illustrates the results of the survey.

(a) State the mode of the distribution.


(b) Find ~ number of adults in the literacy class.
(c) State tile median shoe size.

Answer (a) , : [1]

(b)....•....................... [1]

(c),. "" [1]

400811 482i811 moot

266
Download more resources like this on ECOLEBOOKS.COM
11

15 A

, c
E

In the diagram, AB and Be are adjacent sides of a regular n-sided polygon,


and AB and BD are adjacent sides of a square. The side AB is produced to E.
Given that ABC = 15xo and CBE = 3_xO, calculate the value of

(a) the exterior angle of the n-sided polygon,

(b) n,

(c) reflex CBD.

Answer (a) " [1]

(b) n = [1]

.(c) reflex CBD = " [1]

4001/1 4821/1 N2t11. .


(Tum over

267
Download more resources like12 this on ECOLEBOOKS.COM

16 (a) "Given that f(x) =x/x> 6) and f(l) = k, find the value ofk.

(b) If log3 =m and logS = n, express


(i) log9 in terms of m,
(ii) log2 t in teims of m and n.

Answer (a) .., , [1]

(b)(i) : [1]

(ii}' [1]

400811 402811 N2001

268
Download more resources like this on ECOLEBOOKS.COM
13
Far
17 era ••••
U.
x metres ycm-1

I i I
The diagram shows a rod consisting of two parts x metres and y
centimetres long, where x and y are whole numbers.

(a) Write down, in terms of x and y, an expression for the length of


the rod in centimetres.

(b) If the length of the rod is 244 em, state the value of

(i) x,

(ii) y.

Answer (a). [1]

(b)(i) x = [1]

{ii)y = [J]

4008/14:>2811 N2001
[Turn over

:,.~
I 269
Download more resources like this on ECOLEBOOKS.COM

18

.The diagram shows triangle ABC with AB = 4x em,


BC = 6 em and ABC-= 30°. Given that the area of the
triangle is 15 em", find the value ofx.

[sin 30° = I~; cos 30° = 0,87; tan 30° = 0,58]

Answer x = [3]

40081.1402811 N200l

270
Download more resources like
15 this on ECOLEBOOKS.COM
19 y Fer
ExaniIeI's
10 -Use
.

7
8
-, /

5
.~

-, V
/
~ l7
4 ..•....•••.•.•...........•.•..

3
r--_ V
<a:
r;
2
,V ~
~

V A r-.r--_
1

o
V 1
\\ '\ \

2
I. \ \ \

3
\ \ \\

4
\\\\ \\\\

567
"r~
\ \ \\
v

8 9
-........

10
~
r--
11 12 x

The unshaded region A in the diagram is defined by four inequalities, .


three of which are

y ~ 1,
x + 3y :::;12,
and y:::;x.

(a) Write down the fourth inequality.

(b) Find the least value of 3x + 4y, given that x and y satisfy !he
inequalities.

Answer (a) , [1]

(b) .,.. , - [2]

4008/1 "~28/1 N2001


[Tura ever

271
- ,------~------ -~--
Download more resources like this
16 on ECOLEBOOKS.COM

20 The tabie below gives the changes in the number of jobs in three industries ,""
for the period January 1991 to January 1992 in a certain town.

Industry Change in the number of jobs

Food - 91

Retail - 152
..

Electronics + 186"

(a) Calculate the mean change in the number of jobs for the three
" industries.

(b) Given that there were 150jobs as at January 1991.in the tood industry,
calculate the number of jobs as at January 1992.

(c) Given that there were 286 jobs as at January 1992 in the retail
industry, calculate the number of jobs as at January·1991.

Answer (a) [2]

(b)......................•. [1]

(c) : [1]

400811 4028/1.N2001 "

272
Download more resources like17 this on ECOLEBOOKS.COM

21 ABeD is a rhombus whose diagonals meet at O.

(st) State the size of AOB in degrees.

(b) Given that BD = 10 em and AC = 24 em, calculate AD.

(c) State the radius of the circle that passes through A, 0 and D.

Answer (a) AOB = (I [1]

(b)AD = ~ em [2]

(c) em [1]

400811 4028/1 mOOl


[Turn over

273
Download more resources like this on ECOLEBOOKS.COM
18
Fer
22 &II, ••
U.
T U

p 6cm Q


0(0; 0)

SL...----------'R

W L-----------------------~v
In the diagram, rectangle PQRS is transformed onto rectangle 11JVW
t.
by an enlargement, centre 0 and scale'factor 1 ~Q = 6 em and the area
of rectangle 11JVW is 117 em 2 • ~

(a) Calculate

(i) the length of TIl,


(ii) the area of rectangle PQRS.

(b) Write down the matrix representing the enlargement.

Answer (a) (i) em [1]

(ii) .,em 2 [2] .

(b) ( ) [1]

480lllJ 402811 Nloo)

274
Download more resources like this on ECOLEBOOKS.COM
19

23 It is given that OP = 9a + 5b. Fer


Eramhlr's
Use
(a) Express SOP in terms of a and b.

3h - .
(b) Given that _. a + (h - k)b = OP , find the value of h and the.
4
value ofk.

Answerta) : [1]

(b)h =.................•......

k= [3]

400111402111 NlOOI
[Turnover

275
Download more resources like this on ECOLEBOOKS.COM
20

24 Given the line XY (see answer spaces),

(a) . construct the locus of points that are

(i) 3 em from X,
(ii) 2 em from XV.

(b) mark and label the two possible positions of a point P, above XV,
which are 3 em from X and 2 em from XY.

Answer (a)

X Y

(a)(i) On the diagram. EI]

(ii) On the diagram. [2]

(b) The two positions labelled [I]

400111402111 mOO1

276
Download more resources like this on ECOLEBOOKS.COM
21
.1

: )'b~r":'"
, P.i-..if r;, -t-----7-
,,
-~ 6' f+-.--
~.

4 -- - - .-
'. - .-

I
, .

-'

,-4 -2 o -+- ~
I
2 ~.+++++4--4
,
6 l-X

:v '-x +5
!, !

,,
I!
,
)

,;'. i. ;
j • j
, ,
~ , ,
-x'+6x-5
P(O,-5)
.'
, .~

The diagram shows the graph of the quadratic curve y =- X 2 + 6x - 5.

(a) State

(i) the maximum value ofy,

(ii) the equation of the line of symmetry of the curve.

(b) The curve cuts the y-axis at P(O; -5), find the coordinates ofP 1
the image ofP on the graph, under a reflection in the line of symmetry.

(c) Write down the equation whose roots arc at the intersection of the
graphs y = - x + 5 and y =- X 2 + 6x - 5.

Answer

(c) , ' [2]

400811402811 N2001
[Turn oyer I

277
Download more resources like this on ECOLEBOOKS.COM
22
For
ExamIner's
26 Use

In the diagram, AB is parallel to DC, AD = 3,3 em, DC = 6 em and


BC"" 10 em, ABC = 16° and D:4C = 90°,
(a) Find sin ACD leaving the answer asa proper fraction.

(b) Name an angle which is equal toACD.

(c) - Calculate AC.

[sin16° = 0,28; cos16° = 0,96; tanJ6° = 0,29]

Answer (a) ,....•................. [1]

(b) ........•.......................... [11

(e) .., .: " , em [3]

400811402811 N2001

278
Download more resources like this on ECOLEBOOKS.COM
23
I
27 In this question take 1t
22
to be - ..
7
I~
!Use
II
!
. (a) A solid sphere of radius 10! ern is madeof a metal alloy.
Given that 1ern 3 of the alloy has a mass of 5g, find the mass of the
sphere in kilograms.

4 1
[Volume of sphere = - rtr J
3

(b) The sphere is melted and used, without waste, to make a length of the
wire. Given that the wire is a circular cylinder of radius 0,07 em, find
its length in metres.

Answer (a) kg [3]

(b) m [3]

4008/1 4028/1 N2001


[Turn over
. \ '- ..

279
Download more resources like this on ECOLEBOOKS.COM
MATHEMATICS
NOVEMBER 2001
4008/1-4028/1

POSSIBLE ANSWERS-

19 11 8
1(a) (b) (c) ----
20 36 II
2(a) $40,50 or 4050c (b) 0,031

3(a) 0,012 (b) 0,002 (c) -1.7


4(a) (i) 8 ( II) 81 - (b) 3
5(a) Sm+n or 11 + -5m .b) (r- t)(5r + q)or
(5r + q)(r - t)

6(a) 24 (b) 34 (c) 58


...• ~
7(a) 0,01 (b) 94 25 or 94 .zz: !'
, 100
or 94 'X

8.(a) x<7 or x>7 (b) x6y _ X3y4 (c) 3,71x10


4

9.(a) (b)(i) 16min 275 (ii)18min 235


- 31350r313 IIl'<'

10.(a) 32 (b) 840


11 4 (u) 17 (b) (OUH)' or O'f"lW
12 5 or-5

13 42,2 (b) 147,lO (c) 197,70


14 6 (b) 21 (c) 6
15 30 (b) /12 (c) 240
16 -5 (b) (i) 2m (ii) n-m

17 100 x+y or (b) (i) 2 (ii) 44


y + 100 x
18 2,5 or 2 % or f
2

281
Download more resources like this on ECOLEBOOKS.COM
19. (a) x+y:s::~ (b) 7

20. (a) -19 (b) 5~-1 (c)


. I
438

21 (a) 90 Ib:, 13 (c: 6 Yz or 6,5

22 (a) C)I, 9 (ii) 52 (b)


0
Co~ H )0r

(! ) 0

3
2

h = 12
23. (a) 45a+2Sh or (b) k =7
25h + 45a

24. (a) (i) Circle centre X Oi) Two lines parallel to XI(. one on
radius 3±O,2cm either side, at a distance of 2 em
from XY.

(b) Points of intersection


of (a) (i) and (a) (ii)
above line XY

25. (a) (i) 4 (i i) x-~ (b) (6; -5)


or =X

(c) x2 - 7x + 10 = 0
or (x - 2) (.\"- 5) =0

11 1
26. (a) (b) sAc (a) 5,09 or 5 IT or 5,09
20

27. (a) 24,255 (b) 3 150

"
-' 0'
28. (a) 64,8 (b) (i) ··or"ll (j i) 20 (ii!) 210
5

AM40081·28NOl

282
Download more resources like this on ECOLEBOOKS.COM

ZIMBABWE SCHOOL EXAMINATIONS COUNCIL


General Certificate of Education Ordinary Level

MATHEMATICS 4008/2
PAPER 2

Tuesday 4 DECEMBER 2001 Morning 2 hours 30 minutes


Additional materials
Answer sheet
Geometrical instruments
Graph paper (3sheets)
Mathematical tables
Plain paper (1 sheet)
152730

TIME 2 hours 30 minutes

INSTRUCTIONS TO CANDIDATES

Write your name, Centre number and candidate number in the spaces provided on the answer
paper/answer booklet. .
Answer all questions in Section A and any three questions from Section B
Write your answer on the separate answer paper provided.
If you use more than one sheet of paper, fasten the sheets together.
Electronic calculators must not be used.

All working must be clearly shown. It should be done on the same sheet together as the rest of the
answer .
. Omission of essential working will result in loss of marks.
If the degree of accuracy is not specified in the question and if the answer is not exact, the
answer should be given to three significant figures. Answers in degrees should be given to one
decimal place.

INFORMATION FOR CANDIDATES

The number of marks is given in brackets [ ] at the end of each question or part question.
Mathematical tables may be used to evaluate explicit numerical expressions

This question paper consists of 12 printed pages.

Copyright: Zimbabwe School Examination Council, N2001.

©ZIMSEC N2001 [Turn over

283
Download more resources like this on ECOLEBOOKS.COM
2

Section A [64 marks]

Answer all the questions in this section.

1 (a) Linda spent t of her pocket money and had $20 left. Find the amount
ofwoney she had at first. [3]

(b)

f'
The diagram above represents a rectangular garden 9 m long and 7 m
wide with a path 50 em wide. Calculate the area of the path giving
the answer in square metres. [4]

(c) Express 54 minutes as a percentage of2 hours. [2]

(d) A shop sells a packet of biscuits for$23,46 making a profit of 15% on


the cost price. Calculate the cost price. [3]

400S1'2 N200 1

284
Download more resources like this on ECOLEBOOKS.COM
3

2 (a) Write down


,
(i). the factors of x2 - 4,

(ii) the factors of x2 + 4x + 4 ,

(iii) the L.C.M. of x2 - 4 and x2+4x+ 4. [3]

(b) Solve the equation 9 + 18 = O. [2]


. a

(e) Express -1 + -1 + -1 as a SIn


. gl'frac:
e tion. [2]
s t sf

(d) A formula that can be used for changing degrees'Fahrenheit (F)


to degrees centigrade (C) is

c= 5F-160.
9

(i) Calculate the value of C when F = 95.

(il) Make F the subject of this formula [4]

400812 N2001
[Turn over

285
Download more resources like this on ECOLEBOOKS.COM
4
3

In the diagram, PS = 4,8 ern, SQ = 12 ern, SQR = 36,70 and QRs = 90°.
Calculate

(a) the length of QR, [2]

(b) the length of PR, [3]

(c) the size of the angle marked i', [2]

(d) the area of the triangle PSQ. [2]

4 A municipal swimming pool is 12 m long.o m wide and 1,5 m deep.

(a) Calculate the total internal surface area of the pool. [2]

(b) The municipality employs Harry to paint the pool. One litre of paint
covers 3 m 1 • The paint is sold in 5 litre tins at $850 per tin.

Calculate

(i) Jhe number of tins required,


-
(ii) the cost of the paint. [5]

(c) Harry is paid $432 for painting the pool. Calculate his rate of
pay per hour, if he takes 3 days, working 8 hours a day. [2]

(d) The pool is then filled with water to a depth of 1,25 m. Calculate the
volume, in litres, of water in the pool. [3]

400812 N2001

286
Download more resources like this on ECOLEBOOKS.COM
6

5 (a) ~ is the point (6; 3) and Q is the point (2; 6).

Calculate

(i)
---
PQ,

(ii)
I~· [3]

(b) .
GIven that A = I C I)-3
andB= (02 -2)
-6 '

calculate

..(i) A+B, [2]

(ii) AD, [2]


(iii) the inverse ofB, [2]

(iv) the value ofK, given that

f
A2 = CO
0
0) and A'
10
=K (1
lO
~J. [2]

6 Answer the whole of this question on II sheet of plain paper.

Use ruler and compass only for all constructions and show clearly all the construction
lines and arcs. ;

William is walking straight across a field towards a vertical flagpole. He observes that
When he is at point A, the angle of elevation ofT, the top of the flagpole, is 300 and when
he is at point D, on the same side of the flagpole as A, the angJe of elevation of T is 45°.
The distance between A and B is 20 metres.

(a) . Using a scale of 1 em to represent 4m, construct a single diagram to show


the relative positions of A, B and T. [5]

(b) Using a suitable construction on the same diagram, find the height of the
flagpole. [4]

401)8.'2 N2001
,
[Tum over

287
Download more resources like6 this on ECOLEBOOKS.COM

Section B (36 marks J

Answer three questions in this section.

7 (8) Given the two equations

3x+2y=9 and

2x + 3y = 16 , find

. (i) the value of x + y by adding the 1\\;0 equations,

(il) the value of x - y by subtracting the two equations.

Hence, or otherwise, solve the equations f(;f x and y. [4]

(b)

3x-2 C
O ------------~~-------,
x-3

x+4 B

In the diagram, ABCD is a trapezium in which AB is parallel to DC and


~C=~. .

(i) Given that AB = (x + 4) em, DC::: (3x - 2) em and BC = (x - 3) em,


ind in terms of x, an expression for the area of the trapezium.

Cn) Given that the area of the trapezium is 16 em 2 ., form an equation in 'x
and show that it reduces to 2X2 - 5x - 19 = 0 .:

rJb) Solve this equation and hence find the length of BC correct to 1
decimal place. [8]

400812 moo i

288
Download more resources like this on ECOLEBOOKS.COM
7
8 (a) In a group of five boys, three play soccer and two play hockey.

(i) One boy is chosen at random, write down the probability that he is a
soccer player

(ii) Two boys are chosen at random, calculate the probability that one
plays soccer and the other plays hockey. [ 4]

(b) Answer the whole of this question on a sheet of graph paper.

Two variables P and Q are connected by a law of the form P = kQ, where
k is a constant. The table below shows corresponding values of P and Q

P 0 3 6

Q 0 5 10

(i) U sing a scale of 2 em to represent 1 unit on both axes, with P along


the horizontal axis and Q along the vertical axis, draw a graph to
show the relationship between P and Q. [2]

(ii) Use the graph to find the value of

(a) k, [2]

(b) Q when P = 4. [2]

(iii) Calculate the value of P when Q = 30 [ 2]

. ,

4008/2 N 2001

[Turn over

289
Download more resources like this on ECOLEBOOKS.COM
8

9 T

In the diagram, AB is a diameter of the circle, centre O. TA is the tangent at A


'"
and TAC= 2x0• Find, in terms of x,

(i) CBO,
A

(ii) e~A,
A

(iii) ACO,

(iv) COB. [4]

(b)

~i
\1'I~A
\!,"~

\ 55

\
B

40()S/2 N20l'1

290
Download more resources like this
9 on ECOLEBOOKS.COM

In the diagram, A is 5,5 Ian due north ofB, AC = 3,0 km and RAe = 118°.

Calculate

(i) the bearing of C from A,

(ii) the distance that A is east of C,

,'(iii) the distance BC. f8]

10 ° Ap.swer the whole of this question on a sheet of graph paper.

The following is an incomplete table of values for .the function


y = 12 + 5x, - 2x~ . .

f'
x -2 -11. -1 o· 1 2 3 4 5
2

Y -6 0 p 12 15 q 9 0 -13

(a) Calculate the value ofp and the value of q. [2J

(b) Using a scale of 2 em to represent 1 unit on the x - axis, draw the graph
ofy = 12 + 5x-2x2 for -2 ~ x ~ 5. 4]

(c) Use the graph to

(i) solve the equation 4 + 5x - 2X2 = '0,

(li) fmd the gradient of the curve when x = 3,

(iii) find the area bounded by the curve y = 12 + 5x - 2Xl ,

the x - axis, the y-axis and the line x = 3 ° [6]

400812 N2001
[Turn over

291
Download more resources like this
10 on ECOLEBOOKS.COM

11

-rt"-
t

.11

The triangles ABC, A I B I C I' A 2 B 2 C 2 and the line .f are .shown i11 the diagram.

(a) AAI B I C1 is the image of AABC under a translation. Write down the
column vector of this translation. [1]

(b) Describe completely the transformation that maps AABC onto


AA2B2C2• [3]

(c) AA3B3 C3 (not shown in the diagram) is the image of l~ABC


under a reflection in the line t . Write down

(i) the equation of the line l , .

.(ii) the matrix of Hi is reflection,

(iii) the coordinate" of the points A3 and C3 [5]

-------------------------------- --------- .-.----- ----------


4008/2 N200 I

292
.
9
Download more resources like this on ECOLEBOOKS.COM
A

In the diagram, A is 5,5 km due north of B, AC = 3,0 km and RAe = 118°.

Calculate

(i) the bearing ofC from A,

(ii) the distance that A is east of C,

,'(iii) the distance BC. fS]

10 . ~swer the whole ofthis question on a sheet of graph paper.

The following is an incomplete table of values for .the function'


y = 12 + 5x, - 2x~ . '

~
x -2 -IJ.. -I 0' 1 2 3 4 5
2

12 15 q 9 0 -13
Y -6
° P

(a) Calculate the value of p and the value of q. [2J

. (b) Using a scale of2 em to repr esent 1 unit on the x - axis, draw the graph
of y = 12 + 5x - 2X2 for - 2~x~5. 4]

(c) Use the graph to

(i) solve the equation 4 + 5x - 2X2 = '0,


(ii) find the gradient of the curve when x = 3,

(iii) find the area bounded by the curve y = 12 + 5x - 2X2 ,


the x - axis, the y-axis and the line x = 3. [6]

400812 N200 I
[Turn over

291
Download more resources like this on ECOLEBOOKS.COM
11

(d) IlA .•B .•C.. (not shown in the diagram) is the image of IlABC
under a shear with invariant line y = O. Given that A 4 is the
point (4; 1), write down

(i) the shear factor,

(li) the matrix that represents this transformation. [3]

400812 N200!

[Turn ovei

293
Download more resources like this on ECOLEBOOKS.COM
12

12

T
10.5

A B

In this question take 1t to be 22 .


7

A large solid cone is cut into two parts by a plane parallel to its horizontal circular
base, the upper part being a small solid cone. ,

The figure above represents the solid, with V the vertex, AB the base diameter
, and VO the perpendicular height of the large cone. DC is the base diameter and
VP the perpendicular height of the smaller cone.

Given that AO = DB = 14 em, VO = 10,5 em and VP = 3 em,

calculate

(a) DIt [3]

(b) DI\, [3]

(c) ~ ratio of the volume of the smaller cone to the volume of the
larger cone, giving the answer as a fraction in its lowest terms. [3]

'(d) the curved surface area of the smaller cone. [3]

.12
IVolume of a cone = -1t r h. Curved surface area of a cone = 1tr l .J
3

'( .
400812 N200)

294
Download more resources like this on ECOLEBOOKS.COM

MATHEMATICS NOV 2001

4008/4028/02

POSSIBLE ANSWERS

1. (a) (i) Fraction left 1 - ~ M1


5
J
- -
5
. 5 .M1
Therefore amount left = - x 20
2

= $50 A1

(b) Area of big rectangle =9x7 M1


Area of small rectangle = 8x6 M1
Therefore area of path =9x7 - 8x6 M1
= 15m2 1<1

(c) 54 minutes as a % of 2 hrs =54 x 100 M1


120

=45% A1

(d) $23,46 : 115%

Therefore 100% = 100 x 23,46 M2


115 1

=$20,40 A1

[12]

2. (a) (i) Factors of x2- 4 are x +2 and x-2 81

(ii) Factors of xl+4x+4 are x+2 and x+2 81

(iii) LCM = (x + 2) (x +2) (x - 2) 81

9
(b) - + 18 =0
a

295
------------~----------~----~~---------
Download more resources like this on ECOLEBOOKS.COM

9 + 180 =0

180 =-9 M1

a =-/'2 A1

1 I I
(c) -+-+-
s t sf

f+s+l
= B2
sf

(d) (i) C = 5 x 95 -160 M1


9

=350 A1

(ii) 9C = 5F -160
SF = 9C + 160 M1
F = sc + 160 A1
5

[11]

3.. (a) QR = Cos 36,7 0

12

OR = 12 Cos 36,70 ,
M1

=9,621 em A1

(b) SR = Sin 36 ,
- r
12
$R = 12 Sin36,7° M\1

:.PR =4,8 +12 Sin 36,r M1


= 11.97 em A1

(e) tany = 9,621 M1


11,97

296
Download more resources like this on ECOLEBOOKS.COM

., y :: 38,8° or 38<>47' A1

(d) =so =90°+36,1"


= 126,1"

Area of bf)SQ == %x 12 x 4.8 Sin 126 7 M1

:: 23,09 cm. f\ 1

[9]

4. (a) Internal surface area


= 2 x 5 x 1.5 + 2 x 12 x 1,5 + 12 x 5 M1

= 111 m2 A1

(b) (i) No. of 1 I tins = JJj_ M1


'1
-.J

No. of 51 tins = ~Jl


3x5 M1

== R tins A1

(ii) . Cost of paint = 8 .W: 850 M1

= 55800 A1

(c) Total No. of hours = 3 x 8 .


Rate of pay = 1.~~ M1
3 x8

=$18 A1

(d) Volume of water == 12 x 5 x 1,25 m3 M1

= 12 x 5 x 1,25 x 10001 M1

- :;: 750001 A1

[12]

297
Download more resources like this on ECOLEBOOKS.COM

5. (a) U) PQ = (~ )-(: J

B1

. Ml

=5 A1

- (b) (i) A+B= (3+0 1-2 J


. 1+2 -3-6

= (33 -J
-9)
I B2

(ii) AB= (
3 x 0+ Ix 2 (3 x -2) + (l x -6) J
1xO+(-3x 2) (l x -2)+( -3 x-6

(2 -12)
-6 16
82

(iii) det 8 =(0 x -6) - (-2 x 2)

=4

B-1; .!(-6 2 J B2
. 4 -2 0

298
Download more resources like this on ECOLEBOOKS.COM
(iv) A2 = 10 (0
1
0
I )
1

\11

:. k = 1 000 A1
[11]

/
6.

First draw a horizontal line and mark the points A and S,


Scm apart.
Construct an angle of 30° at A M1
X
. .'. .
Correct construction arcs A1
Construct an angle of 45° at B M1
Correct construction arcs . A1
Accurate ~BT with AB= 5 cm±0,2cm ~1

Construct a perpendicular from T to AS produced. L1


Correct construction arcs C1

299
Download more resources like this on ECOLEBOOKS.COM

Length of perpendicular from T in the range


6,6 -7,0 cm. A1

.Height of flag pole = 6,8 x 5


= 27,2 to,8 m A1

.[9]
7. (a) (i) 3x + 2y=9
+(2x +3\' = 16)
5x +5y= 25

:. x + y=5 81
(ii) x - y =-7 81

x+y = 5
+(x - y = -7)
2x =-2
x =-1 81

,·.y=6 81
(b) (i) Area of trapezium =~(3x-2+x+4) (x-3) M1

(ii) %(3x-2+x-4)(x-3) =16 M1


Yz(4x +2)(x-3) =16
U- 5x-19 =0 A1

-b±.J b2 -4ac
(iv) ~ x= .
2a

= -5±~(-5)2_4(2)(-j9)
2x2

= 5±.JlTi 8181
4 81

= 4,6 and -2,1

OR 4,576 and -2,076 81

300
Download more resources like this on ECOLEBOOKS.COM
8C = 4,576-3
=1,6 81

Alternatively by completing the square,'

(x~~ J Bl

f1i BI

Bl

=4,6 and -2,1 81


[12]

8. (a) (0 P(Soccer player) = ~ 81


5.

(ii) P (1$tsoccer 2nd hockey) 2x'!:_


5 4

. 2 3
P (1st hockey 2nd soccer) = - x -
.54

3 2 2 3
••P(l soccerandl hockey) = -x-+-x-
5 4 5 .4
M2

3
= -
5
A1

(b) Refer to graph below\

(i) (0,0), (3,5) and (6,10) correctly plotted P1


Correct straight line drawn L1

301
Download more resources like this on ECOLEBOOKS.COM
I
(ii) (a) k = -----
gradient

3
=- B2
5

(b) Vertical line at P =4 to graph and across to Q


Axis. M1

Q=6,6 A1

(iii) When Q ;:: 3D,

'"

p =: ~x ,0 Mi"
5
= 18 Al
[12]

302
- .--.~------::------~-----~~~--
Download more resources like this on ECOLEBOOKS.COM

303
Download more resources like this on ECOLEBOOKS.COM

9 (a) .

"-
(i) C B 0 = 2xo (angle in alternatiw 3egment) Bl

(ii) COA= 4xo (exterior angle of ~OCB) B1


(iiij ACO =90° -ao . B1
. (iv) COB = 180°~xo B1
(b) (ij Bearing = 180°+118°

=2980 81
(0) Distance = 3 Sin (180°-118°) M1
=2,649km Ai
(ii) - BC2=32+5,52-2x 3 x 5,5 Cos. 118 M1
=32+5,52+2 x3 x 5,5 Cos (180-118) _M1 .
= 54,74 Ai

BC=~54,74 MI

= 7,399/cm. Al

[12]

304
Download more resources like this on ECOLEBOOKS.COM

10. (a) p~5 B1


q=14
B~ "
Refer to graph
(b) The 7·given pOints and (1%;5) and (2;14) correctly
Plotted.
P3
Correct curve, smooth, not grossly
Thick, paSSing through all 9 points C1
(c) (i) =
line y -8 drawn M1
=
x -2,1 and 4,6 A1
(ii) t~ngent drawn at x =3 M1
gradient -7 = A1
(iii) Area = 40,5 (units)2 82
[12J

305
Download more resources like this on ECOLEBOOKS.COM

E(~F*Y::;_.-~ ; I

V =~-::g_~-t"r~<~ .'-: :~-~:


\.".~ ,...---_ ' -4-~ .•_ ~
I •...•..• _,....

..
_i
1\1

[Xtli-F L:" : _.- . . :.

W
I r ~ I

...•... 1'··--·;.···_· __ ·_-

···-I-•...
i·--+-' -t- ' ,

- : --t=+±-~-;.
..,,~. ·c

1\

• I

306
Download more resources like this on ECOLEBOOKS.COM

(b) Rotation M1

goo anticlockwise or 270°clockwise A1


Centre (-3;4) A1

(c) (I) line L has gradient -1 and intercepts the y axis at 0


=
Therefore equation of 1iDe.4S y -x + 0 or y -x = E1

,. (0 -I)
(ii) Matrix of reflection
-1 0
R2

1)(21 40 J 4J
(iii)
( -10 -O· = (-]
-2
-
0

Coordinates are A3 (-1;-2) Bl


C3 (-4; 0) B1

(d) (0 The x coordinate has been increased bya factor 2


Therefore scale factor: = 2 . 81

(ii) Matrix for the shear is 10 2. ) B2


(

[12]

307
Download more resources like this on ECOLEBOOKS.COM
12. (a)

P
1
1
1
1
17,5 7,5
I
1
I
I
A
14 0

"

tJ.sAOV and DPV are similar

Therefore DP ,. l.L: M1
. 3 10.5

DP = 3x 14 M1
10.5

=4cm A1

(b) DA2= (14-DP)2+ 7,52

2 2
DA=Jl 0 +7,5 M2
=12,5cm AI

308
Download more resources like this on ECOLEBOOKS.COM

(c)
Ratio of volumes= (_3_
1j),5
J MI

-
Ratio of height = -

. =
cr~
T 212
X
.
)
3

MI

8
=- Al
343

(d)
Curved surface' area = 22 x 4 x 5 Ml
- 7

= 62.857 A2

[121

309

You might also like